FULL-TEXT: Select All That Apply NCLEX Practice Quiz (100 Questions)

Updated on
By Matt Vera BSN, R.N.

This is the full-text copy of the 100-item quiz Select All That Apply NCLEX Practice Questions.
Use this page to print a copy of the quiz or export it via PDF.

CREATING PDF. For most modern browsers like Chrome, Safari, Firefox, Edge, you can simply click on File> Print > Save as PDF to create a PDF version of this page.

For more quizzes, please visit Nursing Test Bank and Nursing Practice Questions for Free.

Select All That Apply NCLEX Practice #1

NURSESLABS-SATA-1-001

A patient is admitted to the same-day surgery unit for liver biopsy. Which of the following laboratory tests assesses coagulation? Select all that apply. 

  • A. Partial thromboplastin time.
  • B. Prothrombin time.
  • C. Platelet count.
  • D. Hemoglobin
  • E. Complete Blood Count
  • F. White Blood Cell Count

Correct Answer: A, B, and C.

Prothrombin time, partial thromboplastin time, and platelet count are all included in coagulation studies. The hemoglobin level, though important information prior to an invasive procedure like liver biopsy, does not assess coagulation.

  • Option A: Partial thromboplastin time (PTT) is the time it takes for a patient’s blood to form a clot as measured in seconds. It is used to measure the activity of the intrinsic pathway of the clotting cascade. PTT tests the function of all clotting factors except factor VII (tissue factor) and factor XIII (fibrin stabilizing factor).
  • Option B: Prothrombin time (PT) is one of several blood tests routinely used in clinical practice to evaluate the coagulation status of patients. More specifically, PT is used to evaluate the extrinsic and common pathways of coagulation, which would detect deficiencies of factors II, V, VII, and X, and low fibrinogen concentrations.
  • Option C: Platelet count is being assessed to determine the number of platelets in a sample of the blood as part of a health exam; to screen for, diagnose, or monitor conditions that affect the number of platelets, such as a bleeding disorder, a bone marrow disease, or other underlying conditions.
  • Option D: Hemoglobin is used to evaluate the hemoglobin content of your blood as part of a general health checkup; to screen for and help diagnose conditions that affect red blood cells (RBCs); if there is anemia (low hemoglobin) or polycythemia (high hemoglobin), and to assess the severity of these conditions and to monitor response to treatment.
  • Option E: The complete blood count (CBC) is a group of tests that evaluate the cells that circulate in blood, including red blood cells (RBCs), white blood cells (WBCs), and platelets (PLTs). The CBC can evaluate your overall health and detect a variety of diseases and conditions, such as infections, anemia and leukemia.
  • Option F: WBC count is used to screen for or diagnose a variety of conditions that can affect the number of white blood cells (WBCs), such as an infection, inflammation or a disease that affects WBCs; to monitor treatment of a disorder or to monitor therapy that is known to affect WBCs.

NURSESLABS-SATA-1-002

A patient is admitted to the hospital with suspected polycythemia vera. Which of the following symptoms is consistent with the diagnosis? Select all that apply. 

  • A. Weight loss.
  • B. Prolonged clotting time.
  • C. Hypertension.
  • D. Headaches.
  • E. Polyphagia.
  • F. Pruritus.

Correct Answer: B, C, D, and F.

Polycythemia vera is a condition in which the bone marrow produces too many red blood cells. This causes an increase in hematocrit and viscosity of the blood. Patients can experience headaches, dizziness, and visual disturbances. Bleeding is also a complication, possibly because the platelets are often very large and somewhat dysfunctional. The bleeding can be significant and can occur in the form of nosebleeds, ulcers, frank GI bleeding, hematuria, and intracranial hemorrhage.

  • Option A: Weight loss is not a manifestation of polycythemia vera. Weight loss may result from early satiety or from the increased myeloproliferative activity of the abnormal clone.
  • Option B: Patients with polycythemia vera are at increased risk for thrombosis that may result in CVAs (strokes, brain attacks) or myocardial infarctions (MIs); thrombotic complications are the most common cause of death.
  • Option C: Cardiovascular effects include increased blood pressure and delayed clotting time. Thrombotic complications (1%) include venous thrombosis or thromboembolism and an increased prevalence of stroke and other arterial thrombosis.
  • Option D: Physical complaints can include fatigue, headache, dizziness, tinnitus, vision changes, insomnia, claudication, pruritus, gastritis, and early satiety. Subsequent sludging of blood flow and thrombosis lead to poor oxygen delivery, with symptoms that include headache.
  • Option E: Early satiety can occur in patients with splenomegaly, because of gastric filling being impaired by the enlarged spleen or, rarely, as a symptom of splenic infarction.
  • Option F: Generalized pruritus is caused by histamine release due to an increased number of basophils. Aquagenic pruritus, which occurs during or after a hot shower, is a complaint in 40% of patients. The mechanism is likely from mast cell and basophil degranulation, causing a histamine surge.

NURSESLABS-SATA-1-003

The nurse is teaching the client how to use a metered-dose inhaler (MDI) to administer a Corticosteroid drug. Which of the following client actions indicates that he is using the MDI correctly? Select all that apply.

  • A. The inhaler is held upright.
  • B. Head is tilted down while inhaling the medication.
  • C. Client waits 5 minutes between puffs.
  • D. Mouth is rinsed with water following administration.
  • E. Client lies supine for 15 minutes following administration.

Correct Answer: A & D.

In using a corticosteroid MDI, remove the cap and hold the inhaler upright, stand or sit up straight, shake the inhaler, tilt your head back slightly, put the inhaler in the mouth, press down on the inhaler quickly, breathe in slowly for 3 to 5 seconds, hold the breath for 10 seconds, breathe out slowly, repeat puffs as prescribed, rinse the mouth, and gargle using water or mouthwash after each use.

  • Option A: Keep the chin up and the inhaler upright (not aimed at the roof of the mouth or the tongue). Use a spacer/valve-holding chamber (the best way, useful for all patients) by putting the inhaler into the end with the hole and the mouthpiece end in the mouth. If there is no spacer, hold the inhaler 1 to 2 inches (or two-finger widths) in front of an open mouth.
  • Option B: Head is tilted up during inhalation of the medication. Start breathing in slowly through the mouth and press down on the inhaler one time. If using a spacer or valved-holding chamber, press down on the inhaler before starting to breathe in. Breathe in slowly.
  • Option C: For inhaled quick-relief medicine (like albuterol), wait about 1 minute between puffs. There is no need to wait between puffs for other medicines.
  • Option D: If the client is using this inhaler for a corticosteroid preventer medication, with or without a spacer, rinse the mouth with water and spit after inhaling the last dose to reduce the risk of side effects.
  • Option E: There is no need to lie supine after administration of the medication. If more than one dose is needed, repeat all the steps.

NURSESLABS-SATA-1-004

The nurse is teaching a client with polycythemia vera about potential complications from this disease. Which manifestations would the nurse include in the client’s teaching plan? Select all that apply.

  • A. Hearing loss
  • B. Visual disturbance
  • C. Headache
  • D. Orthopnea
  • E. Gout
  • F. Weight loss

Correct Answer: B, C, D, & E.

Polycythemia vera, a condition in which too many RBCs are produced in the blood serum, can lead to an increase in the hematocrit and hypervolemia, hyperviscosity, and hypertension

  • Option A: Hearing loss is not a manifestation associated with polycythemia vera. Polycythemia vera-related complications and mortality are related to thrombosis, hemorrhage, peptic ulcer disease, myelofibrosis, acute leukemia, or myelodysplastic syndrome (MDS).
  • Option B: Since red blood cells are overproduced in the marrow, this leads to abnormally high numbers of circulating red blood cells (red blood mass) within the blood. Subsequently, the client can experience dizziness, tinnitus, visual disturbances, headaches, or a feeling of fullness in the head.
  • Option C: Thick blood can lead to strokes or tissue and organ damage. Symptoms include lack of energy (fatigue) or weakness, headaches, dizziness, shortness of breath, visual disturbances, nose bleeds, bleeding gums, heavy menstrual periods, and bruising.
  • Option D: The blood thickens and increases in volume, a condition called hyperviscosity. Thickened blood may not flow through smaller blood vessels properly. The client may also experience cardiovascular symptoms such as heart failure (shortness of breath and orthopnea) and increased clotting time.
  • Option E: There are also symptoms of an increased uric acid level such as painful swollen joints (usually the big toe). Gout and kidney stones associated with polycythemia vera occur due to the high turnover of red blood cells, which results in higher-than-normal uric acid production.
  • Option F: Weight loss is not a manifestation associated with polycythemia vera. Weight loss may result from early satiety or from the increased myeloproliferative activity of the abnormal clone.

NURSESLABS-SATA-1-005

Which of the following would be priority assessment data to gather from a client who has been diagnosed with pneumonia? Select all that apply.

  • A. Auscultation of breath sounds
  • B. Auscultation of bowel sounds
  • C. Presence of chest pain.
  • D. Presence of peripheral edema
  • E. Color of nail beds

Correct Answer: A, C, & E.

Physical findings also vary from patient to patient and mainly depend on the severity of lung consolidation, the type of organism, the extent of the infection, host factors, and the existence or nonexistence of pleural effusion.

  • Option A: A respiratory assessment, which includes auscultation of breath sounds is a priority for clients with pneumonia. Low-pitched wheezing (rhonchi) may indicate pneumonia.
  • Option B: Auscultating bowel sounds may be an appropriate assessment, but this is not a priority assessment for the patient with pneumonia. Hyperactive bowel sounds may indicate bowel obstruction, gastroenteritis, or subsiding paralytic ileus. Hypoactive or absent bowel sounds may be present after abdominal surgery, or with peritonitis or paralytic ileus.
  • Option C: Assessing the presence of chest pain is also an important respiratory assessment as chest pain can interfere with the client’s ability to breathe deeply. Grimaces or other expressions of discomfort occurring at the same point in each ventilatory cycle should influence the examiner to identify the origin of that discomfort more precisely.
  • Option D: Assessing for peripheral edema may be an appropriate assessment, but this is not a priority assessment for the patient with pneumonia.
  • Option E: Cyanosis is an indication of decreased perfusion and oxygenation. Alterations and bilateral inconsistencies in color may indicate underlying conditions or injury. With hypoxemia, cyanosis of the extremities or around the mouth may be noted.

NURSESLABS-SATA-1-006

The nurse is teaching a client who has been diagnosed with TB how to avoid spreading the disease to family members. Which statement(s) by the client indicate(s) that he has understood the nurse’s instructions? Select all that apply.

  • A. “I will need to dispose of my old clothing when I return home.”
  • B. “I should always cover my mouth and nose when sneezing.”
  • C. “It is important that I isolate myself from family when possible.”
  • D. “I should use paper tissues to cough in and dispose of them properly.”
  • E. “I can use a regular plate and utensils whenever I eat.”

Correct Answer: B, C, D, & E.

To avoid the spread of the disease, the client diagnosed with tuberculosis should take all the medicines as they’re prescribed and comply with all the doctor’s appointments. Self-isolation should always be strictly followed and also stop yourself from using public transportation.

  • Option A: TB is not spread by sharing glasses, plates, utensils, clothing, sheets, furniture or toilets. These items do not need any special cleaning. TB is not spread by direct physical contact, such as shaking hands, kissing or sex. TB is spread through the air when a person with active TB disease in their lungs coughs, sneezes, sings or talks.
  • Option B: Covering the mouth and nose with a tissue when sneezing is also very important. Hand washing should always be observed right after sneezing or coughing. Inhaling the aerosolized droplets from an infected person is the principal mechanism through which tuberculosis spreads.
  • Option C: Home isolation is when a person must stay at home because they have a contagious disease such as TB. The amount of time needed for home isolation is different for each person. Do not have visitors, especially children and people with weak immune systems. Do not use buses, trains, taxis or airplanes.
  • Option D: Used paper tissues should be disposed of properly. The organism is spread primarily as an airborne aerosol from an individual in the infectious stage of the disease, although transdermal and gastrointestinal (GI) transmission is also possible.
  • Option E: One can only get infected by breathing in TB germs that a person coughs into the air. You cannot get TB from someone’s clothes, drinking glass, eating utensils, handshake, toilet, or other surfaces where a TB patient has been.

Questions and rationale from Nurseslabs.com Feel free to print or share and link back to us! For more practice questions, please visit our Nursing Test Bank [https://nurseslabs.com/nursing-test-bank]

NURSESLABS-SATA-1-007

The nurse is admitting a client with hypoglycemia. Identify the signs and symptoms the nurse should expect. Select all that apply.

  • A. Thirst
  • B. Palpitations
  • C. Diaphoresis
  • D. Slurred speech
  • E. Hyperventilation

Correct Answer: Answer: B, C, & D.

Hypoglycemia is often defined by a plasma glucose concentration below 70 mg/dL; however, signs and symptoms may not occur until plasma glucose concentrations drop below 55 mg/dL. In patients who do not have diabetes, hypoglycemia is uncommon, but when it occurs, there are a few major causes of hypoglycemia: pharmacologic, alcohol, critical illness, counter-regulatory hormone deficiencies, and non-islet cell tumors.

  • Option A: Neurogenic signs and symptoms can either be adrenergic (tremor, palpitations, anxiety) or cholinergic (hunger, diaphoresis, paresthesias). Neurogenic symptoms and signs arise from sympathoadrenal involvement (either norepinephrine or acetylcholine release) in response to perceived hypoglycemia. 
  • Option B: Palpitations, an adrenergic symptom, occur as the glucose levels fall; the sympathetic nervous system is activated and epinephrine and norepinephrine are secreted causing this response.
  • Option C: Diaphoresis is a sympathetic nervous system response that occurs as epinephrine and norepinephrine are released. Low blood sugars can affect activity in the autonomic nervous system (ANS), which is responsible for reactions that people cannot control, such as sweating and digestion.
  • Option D: Slurred speech is a neuroglycopenic symptom; as the brain receives insufficient glucose, the activity of the CNS becomes depressed. These are often called the “warning signs” of hypoglycemia. Lack of glucose to the brain can cause trouble concentrating, changes in vision, slurred speech, lack of coordination, headaches, dizziness and drowsiness.
  • Option E: Ketones are cleared out of the body by the kidneys and expelled through urine. In DKA, ketones build up faster than the kidneys can remove them from the body. This results in a buildup of ketones, which is toxic. The body may try to use the lungs to expel the excess ketones, which causes shortness of breath.

NURSESLABS-SATA-1-008

Which adaptations should the nurse caring for a client with diabetic ketoacidosis expect the client to exhibit? Select all that apply:

  • A. Sweating
  • B. Low PCO2
  • C. Retinopathy
  • D. Acetone breath
  • E. Elevated serum bicarbonate

Correct Answer: B and D.

Metabolic acidosis initiates respiratory compensation in the form of Kussmaul respirations to counteract the effects of ketone buildup, resulting in a lowered PCO2. A fruity odor to the breath (acetone breath) occurs when the ketone level is elevated in ketoacidosis.

  • Option A: Sweating is usually a symptom of hypoglycemia. In diabetic ketoacidosis, insulin deficiency and increased counter-regulatory hormones can lead to increased gluconeogenesis, accelerated glycogenolysis, and impaired glucose utilization. This will ultimately cause worsening hyperglycemia.
  • Option B: The decreased pCO2 that results from this increased respiration returns the pH towards normal but may not be sufficient to achieve a normal pH.
  • Option C: Diabetic retinopathy (DR) is a microvascular disorder occurring due to long term effects of diabetes, leading to vision-threatening damage to the retina, eventually leading to blindness. Uncontrolled diabetes can lead to many ocular disorders like cataract, glaucoma, ocular surface disorders, recurrent stye, non-arteritic anterior ischemic optic neuropathy, diabetic papillopathy, and diabetic retinopathy, out of which diabetic retinopathy is the most common and severe ocular complication.
  • Option D: Patients are often ill-appearing. Kussmaul breathing, which is labored, deep, and tachypneic, may occur. Some providers may appreciate a fruity scent to the patient’s breath, indicative of the presence of acetone.
  • Option E: Acidosis in DKA is due to the overproduction of β-hydroxybutyric acid and acetoacetic acid. At physiological pH, these 2 keto acids dissociate completely, and the excess hydrogen ions bind the bicarbonate, resulting in decreased serum bicarbonate levels.

NURSESLABS-SATA-1-009

When planning care for a client with ulcerative colitis who is experiencing symptoms, which client care activities can the nurse appropriately delegate to an unlicensed assistant? Select all that apply.

  • A. Assessing the client’s bowel sounds
  • B. Providing skin care following bowel movements
  • C. Evaluating the client’s response to antidiarrheal medications
  • D. Maintaining intake and output records
  • E. Obtaining the client’s weight

Correct Answer: Answer: B, D, and E.

Among the tasks that CANNOT be legally and appropriately delegated to nonprofessional, unlicensed assistive nursing personnel, such as nursing assistants, patient care technicians, and personal care aides, include assessments, nursing diagnosis, establishing expected outcomes, evaluating care and any and all other tasks and aspects of care including but not limited to those that entail sterile technique, critical thinking, professional judgment and professional knowledge.

  • Option A: Don’t delegate tasks that require specialized knowledge or complex observations, such as monitoring a patient with chest pain. Even experienced UAP aren’t educationally prepared or licensed to perform such complex tasks.
  • Option B: Examples of tasks which may be assigned include, but are not limited to: clean catheterization technique; simple dressing changes (i.e., clean technique where wound assessment is performed by a licensed nurse and where no wound debridement or packing is involved); suction of chronic tracheostomies (i.e., using clean technique); gastrostomy feedings in established, wound-healed gastrostomies.
  • Option C: As a general rule, don’t delegate the assessment, planning, and evaluation steps of the nursing process. Most nurse practice acts specifically prohibit nurses from delegating initial patient assessments, discharge planning, health education, care planning, triage, and interpretation of assessment data.
  • Option D: A delegatable task is one that doesn’t require nursing judgment. Typically, it’s repetitive—for instance, measuring urine output and vital signs. UAP, licensed practical nurses (LPNs), and licensed vocational nurses (LVNs) can collect patient data, but only the registered nurse can interpret data.
  • Option E: RNs may continue to assign to unlicensed assistive personnel those activities which unlicensed assistive personnel have traditionally performed in the delivery of patient care. Examples include but are not limited to: bathing, feeding, ambulating, vital signs, weight, assistance with elimination, maintaining a safe environment.

NURSESLABS-SATA-1-010

Which of the following nursing diagnoses would be appropriate for a client with heart failure? Select all that apply.

  • A. Ineffective tissue perfusion related to decreased peripheral blood flow secondary to decreased cardiac output.
  • B. Activity intolerance related to increased cardiac output.
  • C. Decreased cardiac output related to structural and functional changes.
  • D. Impaired gas exchange related to decreased sympathetic nervous system activity.

Correct Answer: A and C.

HF is a result of structural and functional abnormalities of the heart tissue muscle. Heart failure results from changes in the systolic or diastolic function of the left ventricle. 

  • Option A: The heart muscle becomes weak and does not adequately pump the blood out of the chambers. As a result, blood pools in the left ventricle and backs up into the left atrium, and eventually into the lungs. Therefore, greater amounts of blood remain in the ventricle after contraction thereby decreasing cardiac output. In addition, this pooling leads to thrombus formation and ineffective tissue perfusion because of the decrease in blood flow to the other organs and tissues of the body. 
  • Option B: Typically, these clients have an ejection fraction of less than 50% and poorly tolerate activity. Activity intolerance is related to a decrease, not increase, in cardiac output. 
  • Option C: The heart fails to pump enough blood to meet the metabolic needs of the body. The blood flow that supplies the heart is also decreased therefore decrease in cardiac output occurs, blood then is insufficient and making it difficult to circulate the blood to all parts of the body thus may cause altered heart rate and rhythm, weakness, and paleness.
  • Option D: Gas exchange is impaired. However, the decrease in cardiac output triggers compensatory mechanisms, such as an increase in sympathetic nervous system activity.

NURSESLABS-SATA-1-011

When caring for a client with a central venous line, which of the following nursing actions should be implemented in the plan of care for chemotherapy administration? Select all that apply.

  • A. Verify patency of the line by the presence of a blood return at regular intervals.
  • B. Inspect the insertion site for swelling, erythema, or drainage.
  • C. Administer a cytotoxic agent to keep the regimen on schedule even if blood return is not present.
  • D. If unable to aspirate blood, reposition the client, and encourage the client to cough.
  • E. Contact the health care provider about verifying placement if the status is questionable.

Correct Answer: A, B, D, and E.

A major concern with the intravenous administration of cytotoxic agents is vessel irritation or extravasation. In order to avoid additional chemotherapy adverse effects, every effort should be made to minimize the complications of chemotherapy administration. All the oncology team members share responsibility to ensure the safe administration of chemotherapy.

  • Option A: The Oncology Nursing Society and hospital guidelines require frequent evaluation of blood return when administering vesicant or non-vesicant chemotherapy due to the risk of extravasation. These guidelines apply to peripheral and central venous lines. 
  • Option B: Chemotherapy extravasation is manifested by a wide range of symptoms that can be mild and can present as an acute burning pain, swelling, at the infusion site. Symptoms vary according to the amount and concentration of extravasated drugs. Pain and erythema, induration and skin discoloration progresses over a few days and weeks, and may progress to blister formation. Unlike flare reaction and vessel irritation, extravasation is usually manifested with no or minimal blood return at the infusion site.
  • Option C: In case of chemotherapy extravasation and as soon as the patient complains of pain or swelling, the first step should be immediate cessation of the infusion while keeping the cannula or port needle in place. This is followed by attempts at aspiration of the chemotherapeutic agent and removing the cannula or port needle.
  • Option D: In addition, central venous lines may be long-term venous access devices. Thus, difficulty drawing or aspirating blood may indicate the line is against the vessel wall or may indicate the line has occlusion. Having the client cough or move position may change the status of the line if it is temporarily against a vessel wall. 
  • Option E: Occlusion warrants a more thorough evaluation via x-ray study to verify placement if the status is questionable and may require a declotting regimen. Any local incidence of extravasation should be reported. While documentation may differ among institutions, certain items remain essential and should be documented for every incident.

NURSESLABS-SATA-1-012

A 20-year old college student has been brought to the psychiatric hospital by her parents. Her admitting diagnosis is borderline personality disorder. When talking with the parents, which information would the nurse expect to be included in the client’s history? Select all that apply.

  • A. Impulsiveness
  • B. Lability of mood
  • C. Ritualistic behavior
  • D. Psychomotor retardation
  • E. Self-destructive behavior

Correct Answer: A, B, and E.

Bipolar affective disorder is a chronic and complex disorder of mood that is characterized by a combination of manic (bipolar mania), hypomanic and depressive (bipolar depression) episodes, with substantial subsyndromal symptoms that commonly present between major mood episodes.

  • Option A: With BPD, the client may engage in harmful, sensation-seeking behaviors, especially when upset. The client may impulsively spend money he or she can’t afford, binge eat, drive recklessly, shoplift, engage in risky sex, or overdo it with drugs or alcohol. 
  • Option B: Unstable emotions and moods are also common with BPD. Affect is often heightened, intense, and extremely labile. Implicit with the affective lability of mania are hyperactivity and severe mobility. When presenting in a depressive state, the patient will report a sad or elegiac mood, while expressing a congruent affect (often tearful).
  • Option C: Ritualistic behavior is common in clients with Obsessive-Compulsive Disorder (OCD). To reduce the anxiety and distress associated with these thoughts, the patient may employ compulsions or rituals. These rituals may be personal and private, or they may involve others to participate; the rituals are to compensate for the ego-dystonic feelings of the obsessional thoughts and can cause a significant decline in function.
  • Option D: Psychomotor retardation occurs commonly during depressive episodes of bipolar disorder as well as major depressive disorder. Major depressive disorder is diagnosed when an individual has a persistently low or depressed mood, anhedonia or decreased interest in pleasurable activities, feelings of guilt or worthlessness, lack of energy, poor concentration, appetite changes, psychomotor retardation or agitation, sleep disturbances, or suicidal thoughts.
  • Option E: Extremely depressed patients demonstrate avolition and abulia–lack of willpower. It has been postulated that suicidal ideation originates during the depressive phases and is made manifest upon the transition to baseline or a subsequent manic state. Manic patients are threatening and assaultive.

NURSESLABS-SATA-1-013

When assessing a client diagnosed with impulse control disorder, the nurse observes violent, aggressive, and assaultive behavior. Which of the following assessment data is the nurse also likely to find? Select all that apply.

  • A. The client functions well in other areas of his life.
  • B. The degree of aggressiveness is out of proportion to the stressor.
  • C. The violent behavior is most often justified by the stressor.
  • D. The client has a history of parental alcoholism and chaotic, abusive family life.
  • E. The client has no remorse about the inability to control his anger.

Correct Answer: A, B, and D.

Impulse control disorders are pervasive and often life long manifestations of disabling behavioral patterns. Unchecked, these disorders can result in deleterious outcomes for those afflicted. Disinhibited psychopathology has precipitated the nosologic identification of ‘impulse control disorders’ (ICD), in DSM 5. Those falling under the taxon of ICD experience “failure to resist an impulse, temptation, or drive to perform an act that is harmful to the other person or others.”

  • Option A: A client with an impulse control disorder who displays violent, aggressive, and assaultive behavior generally functions well in other areas of his life. Between explosive episodes, these patients will demonstrate appropriate behavior; however, upon exposure to minimal adversity, these patients will respond with violent, disproportionate tantrums, which may seem “out of character.” Incidentally, the rapidity of the escalation is mirrored, temporally, by the de-escalation.
  • Option B: The degree of aggressiveness is typically out of proportion with the stressor. The patient may feel like a hapless bystander, victim to his impulses. Most importantly, these behavior patterns are extreme and inappropriate when contrasted with those of similar biological and developmental age, resulting in severe psychosocial and functional impairments. 
  • Option C: The disorders encompassed within impulse control disorder (ICD) are identified as externalizing disorders, as these individuals express hostility and resentment externally, made manifest by conflicts with others; whereas, those with internalizing disorders direct their distress inwardly onto themselves, ego-dystonically.
  • Option D: Such a client commonly has a history of parental alcoholism and a chaotic family life, and often verbalizes sincere remorse and guilt for the aggressive behavior. Social factors implicated in the development of ICD include low socioeconomic status, community violence, lack of structure, neglect, abusive environment, and deviant peer relations.
  • Option E: Antisocial personality disorder causes people to act without thinking how they’re affecting others. Someone with ASPD may break rules or laws. They often show no remorse and take no responsibility. Psychotherapy and certain medications may help people with ASPD.

 NURSESLABS-SATA-1-014

A nurse is caring for a middle-aged client who has undergone hemicolectomy for colon cancer. The client has two children. Which concepts about families should the nurse consider when providing care for this client? Select all that apply.

  • A. Illness in one family member can affect all members.
  • B. Family roles do not change because of illness.
  • C. A family member may perform more than one role at a time.
  • D. Children typically are not affected by adult illness.
  • E. The effects of an illness on a family depend on the stage of the family’s life cycle.
  • F. Changes in sleeping and eating patterns may be signs of stress in a family.

Correct Answer: A, C, E, and F.

Quality of life (QoL) of individuals is closely related to the QoL of those around them, including partners or parents. Therefore, any chronic illness carries the potential to impact on the life of the family.

  • Option A: Illness in one family member can affect all family members, even children. Family members suffer greatly from the emotional effects of living with, and caring for, a relative with a disease, with the impact of some diseases being felt by every member of the family.
  • Option B: Family members of patients experience a negative effect on their family relationships, both between the relative and the patient, and between other members of the family as a result of the patient’s illness.
  • Option C: Each member of a family may have several roles to perform. A middle-aged client, for example, may have the roles of father/mother, husband/wife, wage earner, child care provider, and housekeeper. When one family member cannot fulfill a role because of illness, the roles of the other family members are affected. 
  • Option D: Most chronic diseases have similar effects on family members including psychological and emotional functioning, disruption of leisure activities, effect on interpersonal relationships, and financial resources.
  • Option E: Families move through certain predictable life cycles (such as birth of a baby, a growing family, adult children leaving home, and grandparenting). The impact of illness on the family depends on the stage of the life cycle as family members take on different roles and the family structure changes. 
  • Option F: Illness produces stress in families; changes in eating and sleeping patterns are signs of stress. The psychological distress felt by family members often results from their feelings of helplessness and lack of control. Many different emotions are mentioned by family members; guilt, anger, worry, upset, frustration, embarrassment, despair, loss, relief.

Questions and rationale from Nurseslabs.com Feel free to print or share and link back to us! For more practice questions, please visit our Nursing Test Bank [https://nurseslabs.com/nursing-test-bank]

NURSESLABS-SATA-1-015

The nurse is monitoring a client receiving peritoneal dialysis and the nurse notes that a client’s outflow is less than the inflow. Select actions that the nurse should take. Select all that apply. 

  • A. Place the client in good body alignment
  • B. Check the level of the drainage bag
  • C. Contact the physician
  • D. Check the peritoneal dialysis system for kinks
  • E. Reposition the client to his or her side

Correct Answer: A, B, D, and E.

Outflow (one-way) is the most common type of obstruction. This obstruction is caused by the closeness of the distal portion of the catheter to the omentum or intestine, which allows infusion of the solution, but little-to-no outflow.

  • Options A and E: If outflow drainage is inadequate, the nurse attempts to stimulate outflow by changing the client’s position. Turning the client to the other side or making sure that the client is in good body alignment may assist with outflow drainage. 
  • Option B: The drainage bag needs to be lower than the client’s abdomen to enhance gravity drainage. Using a titanium weight at the end of the catheter, front-loading , or laparoscopic salvage of the catheter with reposition and securing the internal tip of the catheter in the true pelvis with a stitch can prevent or correct this complication.
  • Option C: There is no reason to contact the physician. Omental wrapping can occur at any time after catheter insertion. Conservative therapy with enemas, change in position and ambulation often remedy this problem.
  • Option D: The connecting tubing and the peritoneal dialysis system is also checked for kinks or twisting and the clamps on the system are checked to ensure that they are open. Persistent obstruction may require catheter manipulation with reposition or replacement in extreme cases.

NURSESLABS-SATA-1-016

The nurse is caring for a hospitalized client who has chronic renal failure. Which of the following nursing diagnoses are most appropriate for this client? Select all that apply.

Correct Answer: A, B, C, and E.

Appropriate nursing diagnoses for clients with chronic renal failure include excess fluid volume related to fluid and sodium retention; imbalanced nutrition, less than body requirements related to anorexia, nausea, and vomiting; and activity intolerance related to fatigue. 

  • Option A: Renal disorder impairs glomerular filtration that results in fluid overload. With fluid volume excess, hydrostatic pressure is higher than the usual pushing excess fluids into the interstitial spaces. Since fluids are not reabsorbed at the venous end, fluid volume overloads the lymph system and stays in the interstitial spaces.
  • Option B: Due restricted foods and prescribed dietary regimen, an individual experiencing renal problem cannot maintain ideal body weight and sufficient nutrition. At the same time patients may experience anemia due to decreased erythropoietic factors that cause decrease in production of RBC causing anemia and fatigue.
  • Option C: Assess the extent of weakness, fatigue,ability to participate in active and passive activities. This provides information about the impact of activities on fatigue and energy reserves.
  • Option D: Gas exchange is not impaired in CRF. Instead, there is a dysfunction in renal tissue perfusion. For optimal cell functioning the kidney excrete potentially harmful nitrogenous product- urea, creatinine, and uric acid, but because of the loss of kidney excretory functions there is impaired excretion of nitrogenous waste product causing an increase in laboratory results of BUN, creatinine, and uric acid level.
  • Option E: Pain is a discomfort that is caused by the stimulation of the nerve endings. Any trauma that the kidney experiences (by any causes or factors) perceived by the body as a threat, the body releases cytokine and prostaglandin causing pain which is felt by the patient at his flank area.

NURSESLABS-SATA-1-017

The nurse is assessing a child diagnosed with a brain tumor. Which of the following signs and symptoms would the nurse expect the child to demonstrate? Select all that apply.

  • A. Head tilt
  • B. Vomiting
  • C. Polydipsia
  • D. Lethargy
  • E. Increased appetite
  • F. Increased pulse

Correct Answer: A, B, and D.

Head tilt, vomiting, and lethargy are classic signs assessed in a child with a brain tumor. Clinical manifestations are the result of location and size of the tumor. Pediatric brain tumors are the most common type of solid childhood cancer and only second to leukemia as a cause of pediatric malignancies.

  • Option A: A brain tumour at the back of the head can cause the child to have a stiff neck. They may develop a head tilt. This is where the child holds their head or neck in an unusual way, such as at an awkward angle or in a twisted position. The child may develop what is called a ‘wry neck’.
  • Option B: As a brain tumor grows larger, it takes up more and more space within the skull, thereby increasing intracranial pressure. This increased pressure can lead to feelings of nausea. Nausea and vomiting also can occur when a tumor develops in a particular area of the brain. The cerebellum, for instance, is the part of the brain that controls balance, so if a brain tumor presses against the cerebellum, the person could end up feeling dizzy and nauseated.
  • Option C: Primary polydipsia can be categorized into two types. They are psychogenic polydipsia and dipsogenic polydipsia. Psychogenic polydipsia is a condition seen usually in patients with psychiatric diseases. Dipsogenic polydipsia is seen in patients with hypothalamic conditions and in otherwise healthy individuals who are drinking excessive amounts of fluids in today’s context of a healthier lifestyle.
  • Option D: In neonates and older children, the clinical presentation depends on the site of tumor involvement. Supratentorial tumors may present with limb weakness, convulsions, and altered level of consciousness.
  • Option E: The location of the tumor can make it difficult for the patient to eat, or cause symptoms that disturb the appetite. For instance, tumors located in the base of the skull can lead to swallowing difficulties. And depression and anxiety, common in people with spinal cord and brain tumors, can affect the diet, too.
  • Option F: Sinus bradycardia (sinus rhythm of less than 60 bpm), which can be a normal phenomenon, has been only sporadically documented in the literature as the first signs of a brain tumor.

NURSESLABS-SATA-1-018

The nurse is caring for a client with a T5 complete spinal cord injury. Upon assessment, the nurse notes flushed skin, diaphoresis above the T5, and blood pressure of 162/96. The client reports a severe, pounding headache. Which of the following nursing interventions would be appropriate for this client? Select all that apply.

  • A. Elevate the HOB to 90 degrees
  • B. Loosen constrictive clothing
  • C. Use a fan to reduce diaphoresis
  • D. Assess for bladder distention and bowel impaction
  • E. Administer antihypertensive medication
  • F. Place the client in a supine position with legs elevated

Correct Answer: A, B, D, & E.

The client has signs and symptoms of autonomic dysreflexia. The potentially life-threatening condition is caused by an uninhibited response from the sympathetic nervous system resulting from a lack of control over the autonomic nervous system. 

  • Option A: The nurse should immediately elevate the HOB to 90 degrees and place extremities dependently to decrease venous return to the heart and increase venous return from the brain. 
  • Option B: Because tactile stimuli can trigger autonomic dysreflexia, any constrictive clothing should be loosened. Removal of constrictive clothing and vascular support also promotes venous pooling to help lower BP.
  • Option C: A fan shouldn’t be used because cold drafts may trigger autonomic dysreflexia. Identify and monitor precipitating risk factors like temperature extremes or drafts.
  • Option D: The nurse should also assess for distended bladder and bowel impaction, which may trigger autonomic dysreflexia, and correct any problems. Removing noxious stimuli usually terminates episodes and may prevent more serious autonomic dysreflexia.
  • Option E: Elevated blood pressure is the most life-threatening complication of autonomic dysreflexia because it can cause stroke, MI, or seizures. If removing the triggering event doesn’t reduce the client’s blood pressure, IV antihypertensives should be administered.
  • Option F: Elevate head of bed to 45-degree angle or place patient in sitting position. This lowers BP to prevent intracranial hemorrhage, seizures, or even death. Placing tetraplegic in sitting position automatically lowers BP.

NURSESLABS-SATA-1-019

The nurse is evaluating the discharge teaching for a client who has an ileal conduit. Which of the following statements indicates that the client has correctly understood the teaching? Select all that apply.

  • A. “If I limit my fluid intake I will not have to empty my ostomy pouch as often.”
  • B. “I can place an aspirin tablet in my pouch to decrease odor.”
  • C. “I can usually keep my ostomy pouch on for 3 to 7 days before changing it.”
  • D. “I must use a skin barrier to protect my skin from urine.”
  • E. “I should empty my ostomy pouch of urine when it is full.”

Correct Answer: C & D.

The client with an ileal conduit must learn self-care activities related to care of the stoma and ostomy appliances. The ileal conduit is not continent because of its small size. Urine is not collected and held in the pouch but continuously flows out of the stoma. An ileal conduit requires the client to wear an external urostomy bag that adheres to the skin around the stoma and collects urine. 

  • Option A: The client should be taught to increase fluid intake to about 3,000 ml per day and should not limit intake. Adequate fluid intake helps to flush mucus from the ileal conduit. 
  • Option B: Aspirin should not be used as a method of odor control because it can be an irritant to the stoma and lead to ulceration. Devrom is a chewable odor eliminator that contains bismuth subgallate, which works to neutralize odors from stool and flatulence. Always make sure to talk to a doctor before using any medications and understand any side effects that may occur.
  • Option C: The ostomy appliance should be changed approximately every 3 to 7 days and whenever a leak develops. If there is itching and burning around the stoma, it is a sign to change the pouching system and clean the surrounding skin.
  • Option D: A skin barrier is essential to protecting the skin from the irritation of the urine. Cleaning the stoma and skin with water is enough. If the client wishes to use soap, rinse well as soap might affect the adhesiveness of the skin barrier. Pat the skin dry before putting on the skin barrier and pouch. 
  • Option E: The ostomy pouch should be emptied when it is one-third to one-half full to prevent the weight from pulling the appliance away from the skin. The nighttime drainage bag and leg bag should be replaced every 30 days, with a limited number covered by insurance. The drainage bags should be cleansed, especially when switching from a leg bag to a night bag. Both should be rinsed with warm water after each use.

NURSESLABS-SATA-1-020

A nurse is assisting in performing an assessment on a client who suspects that she is pregnant and is checking the client for probable signs of pregnancy. Select all probable signs of pregnancy.

  • A. Uterine enlargement
  • B. Fetal heart rate detected by a nonelectric device
  • C. Outline of the fetus via radiography or ultrasound
  • D. Goodell’s sign
  • E. Braxton Hicks contractions
  • F Ballottement

Correct Answer: A, D, E, & F.

Probable signs of pregnancy are those signs commonly noted by the physician upon examination of the patient. These signs include uterine changes, abdominal changes, cervical changes, basal body temperature, positive pregnancy test by physician, and fetal palpation.

  • Option A: The uterine increases in width and length approximately five times its normal size. Its weight increases from 50 grams to 1,000 grams. By the twelfth week, the uterus rises above the symphysis pubis and it should reach the xiphoid process by the 36th week of pregnancy.
  • Option B: Fetal heart sounds are positive signs of pregnancy. The fetal heart begins beating by the 24th day following conception. It is audible with a doppler by 10 weeks of pregnancy and with a fetoscope after the 16th week (see figure 3-5). It is not to be confused with uterine souffle or swishlike tone from pulsating uterine arteries. The normal fetal heart rate is 120 to 160 beats.
  • Option C: Confirmation of fetal outline through ultrasound is a positive sign of pregnancy. The gestation sac can be seen and photographed. An embryo as early as the 4th week after conception can be identified. The fetal parts begin to appear by the 10th week of gestation.
  • Option D: The cervix is normally firm like the cartilage at the end of the nose. The Goodell’s sign is when there is marked softening of the cervix. This is present at 6 weeks of pregnancy.
  • Option E: These contractions will, generally, cease with walking or other forms of exercise. The Braxton-Hicks contractions are distinct from contractions of true labor by the fact that they do not cause the cervix to dilate and can usually be stopped by walking.
  • Option F: This is demonstrated during the bimanual exam at the 16th to 20th week. Ballottement is when the lower uterine segment or the cervix is tapped by the examiner’s finger and left there, the fetus floats upward, then sinks back and a gentle tap is felt on the finger. This is not considered diagnostic because it can be elicited in the presence of ascites or ovarian cysts.

Questions and rationale from Nurseslabs.com Feel free to print or share and link back to us! For more practice questions, please visit our Nursing Test Bank [https://nurseslabs.com/nursing-test-bank]

NURSESLABS-SATA-1-021

A nurse is monitoring a pregnant client with pregnancy induced hypertension who is at risk for Preeclampsia. The nurse checks the client for which specific signs of Preeclampsia? Select all that apply.

  • A. Elevated blood pressure
  • B. Negative urinary protein
  • C. Facial edema
  • D. Increased respirations

Correct Answer: A & C.

The three classic signs of preeclampsia are hypertension, generalized edema, and proteinuria. Increased respirations are not a sign of preeclampsia. Preeclampsia is a hypertensive disorder in pregnancy-related to 2% to 8% of pregnancy-related complications worldwide. It results in 9% to 26% of maternal deaths in low-income countries and 16% in high-income countries. Preeclampsia is defined as new-onset hypertension.

  • Option A: The parameters for initial identification of preeclampsia are specifically defined as a systolic blood pressure of 140 mm Hg or more or diastolic blood pressure of 90 mm Hg or more on two occasions at least 4 hours apart; or shorter interval timing of systolic blood pressure of 160 mm Hg or more or diastolic blood pressure of 110 mm Hg or more, all of which must be identified after 20 weeks of gestation.
  • Option B: Although elevated blood pressure with accompanying proteinuria is typically thought to be required for the diagnosis of preeclampsia, it may not be present in several cases. In such cases, where the absence of proteinuria and new-onset hypertension is discovered, other new-onset symptoms such as thrombocytopenia, renal insufficiency, pulmonary edema, impaired liver function, or new-onset headache with or without visual disturbance may be used for diagnosis.
  • Option C: Overall evaluation for edema should also be completed, specifically evaluating areas of dependent (gravity-related) edema like the lower extremities or independent edema, such as in the face or hands.
  • Option D: Shortness of breath and a perceived increase in swelling, both worsening from baseline pregnancy-related symptoms, may also be reported. Suppose patients present with shortness of breath, auscultation, and percussion of lungs should be undertaken to examine for pulmonary disturbances.

NURSESLABS-SATA-1-022

A nurse is caring for a pregnant client with severe preeclampsia who is receiving IV magnesium sulfate. Select all nursing interventions that apply in the care for the client.

  • A. Monitor maternal vital signs every 2 hours
  • B. Notify the physician if respirations are less than 18 per minute.
  • C. Monitor renal function and cardiac function closely
  • D. Keep calcium gluconate on hand in case of a magnesium sulfate overdose
  • E. Monitor deep tendon reflexes hourly
  • F. Monitor I and O’s hourly
  • G. Notify the physician if urinary output is less than 30 ml per hour.

Correct Answer: C, D, E, F, & G.

Preeclampsia is a hypertensive disease that occurs during pregnancy. This disease encompasses 2% to 8% of pregnancy-related complications, greater than 50,000 maternal deaths, and over 500,000 fetal deaths worldwide.

  • Option A: Monitor and record vital signs (blood pressure, pulse, respirations, O2 saturation) every 1 hour x’s 8 hours after maintenance infusion is started and vital signs for bolus infusion are complete.
  • Option B: When caring for a client receiving magnesium sulfate therapy, the nurse would monitor maternal vital signs, especially respirations, every 30-60 minutes and notify the physician if respirations are less than 12, because this would indicate respiratory depression. 
  • Option C: Cardiac and renal function is monitored closely. The patient with PIH does not display the normal cardiovascular response to pregnancy (left ventricular hypertrophy, increase in plasma volume, vascular relaxation with decreased peripheral resistance).
  • Option D: Calcium gluconate is kept on hand in case of magnesium sulfate overdose, because calcium gluconate is the antidote for magnesium sulfate toxicity. 
  • Option E: Deep tendon reflexes are assessed hourly. A therapeutic level of MgSO4is achieved with serum levels of 4.0–7.5 mEq/L or 6–8 mg/dL. Adverse/toxic reactions develop above 10–12 mg/dL, with loss of DTRs occurring first, respiratory paralysis between 15–17 mg/dL, or heart block occurring at 30–35 mg/dL.
  • Option F: Monitor intake and output. Note urine color, and measure specific gravity as indicated. Urine output is a sensitive indicator of circulatory blood volume. Oliguria and specific gravity of 1.040 indicate severe hypovolemia and kidney involvement. Administration of magnesium sulfate (MgSO4)may cause transient increase in output.
  • Option G: The urine output should be maintained at 30 ml per hour because the medication is eliminated through the kidneys. Urine output should be at least 30 mL/hour while administering magnesium sulfate. If less, notify the provider of decreased urine output.

NURSESLABS-SATA-1-023

When interpreting an ECG, the nurse would keep in mind which of the following about the P wave? Select all that apply.

  • A. Reflects electrical impulse beginning at the SA node
  • B. Indicated electrical impulse beginning at the AV node
  • C. Reflects atrial muscle depolarization
  • D. Identifies ventricular muscle depolarization
  • E. Has duration of normally 0.11 seconds or less

Correct Answer: A, C, & E.

The P wave and PR segment is an integral part of an electrocardiogram (ECG). It represents the electrical depolarization of the atria of the heart. It is typically a small positive deflection from the isoelectric baseline that occurs just before the QRS complex.

  • Options A and B: In a client who has had an ECG, the P wave represents the activation of the electrical impulse in the SA node, which is then transmitted to the AV node. The P wave represents the electrical depolarization of the atria.  In a healthy person, this originates at the sinoatrial node (SA node) and disperses into both left and right atria. 
  • Options C and D: In addition, the P wave represents atrial muscle depolarization, not ventricular depolarization. Depolarization of the right atrium is responsible for the early part of the P wave, and depolarization of the left atrium is responsible for the middle and terminal portions of the P wave. 
  • Option E: The normal duration of the P wave is 0.11 seconds or less in duration and 2.5 mm or more in height. In a normal EKG, the P-wave precedes the QRS complex. It looks like a small bump upwards from the baseline. The amplitude is normally 0.05 to 0.25mV (0.5 to 2.5 small boxes). Normal duration is 0.06-0.11 seconds (1.5 to 2.75 small boxes).

NURSESLABS-SATA-1-024 

When caring for a client with a central venous line, which of the following nursing actions should be implemented in the plan of care for chemotherapy administration? Select all that apply.

  • A. Verify patency of the line by the presence of a blood return at regular intervals.
  • B. Inspect the insertion site for swelling, erythema, or drainage.
  • C. Administer a cytotoxic agent to keep the regimen on schedule even if blood return is not present.
  • D. If unable to aspirate blood, reposition the client, and encourage the client to cough.
  • E. Contact the health care provider about verifying placement if the status is questionable.

Correct Answer: A, B, D, & E.

A major concern with intravenous administration of cytotoxic agents is vessel irritation or extravasation. In order to avoid additional chemotherapy adverse effects, every effort should be made to minimize the complications of chemotherapy administration. All the oncology team members share responsibility to ensure the safe administration of chemotherapy.

  • Option A: The Oncology Nursing Society and hospital guidelines require frequent evaluation of blood return when administering vesicant or non-vesicant chemotherapy due to the risk of extravasation. These guidelines apply to peripheral and central venous lines.
  • Option B: Chemotherapy extravasation is manifested by a wide range of symptoms that can be mild and can present as an acute burning pain, swelling, at the infusion site. Symptoms vary according to the amount and concentration of extravasated drugs. Pain and erythema, induration and skin discoloration progresses over a few days and weeks, and may progress to blister formation. Unlike flare reaction and vessel irritation, extravasation is usually manifested with no or minimal blood return at the infusion site.
  • Option C: In case of chemotherapy extravasation and as soon as the patient complains of pain or swelling, the first step should be immediate cessation of the infusion while keeping the cannula or port needle in place. This is followed by attempts at aspiration of the chemotherapeutic agent and removing the cannula or port needle.
  • Option D: In addition, central venous lines may be long-term venous access devices. Thus, difficulty drawing or aspirating blood may indicate the line is against the vessel wall or may indicate the line has occlusion. Having the client cough or move position may change the status of the line if it is temporarily against a vessel wall. 
  • Option E: Occlusion warrants more thorough evaluation via x-ray study to verify placement if the status is questionable and may require a declotting regimen. Any local incidence of extravasation should be reported. While documentation may differ among institutions, certain items remain essential and should be documented for every incident.

NURSESLABS-SATA-1-025

A nurse is assessing a newly admitted client. In the family assessment, who should be considered as part of the client’s family? Select all that apply.

  • A. People related by blood or marriage
  • B. People whom the client views as family
  • C. People who live in the same house
  • D. People whom the nurse thinks are important to the client
  • E. People of the same racial background who live in the same house as the client
  • F. People who provide for the physical and emotional needs of the client

Correct Answer: B & F.

The term “family” is difficult to define. The mid 20th century concept of family, with heterosexual parents and offspring living under the same roof is now seldom used, and many authors now consciously use a wider definition of family. The dynamics between family members are constantly evolving and there is evidence of many diverse family types in modern western European society.

  • Option A: Poston et al. define family as “people who think of themselves as part of the family, whether by blood or marriage or not, and who support and care for each other on a regular basis”, and this definition is thought to acknowledge the diverse social arrangements that may constitute a family. 
  • Option B: When providing care to a client, the nurse should consider family members to be all the people whom the client views as family. Rather than simply defining family by a dictionary definition, each individual should look to define a family by their own standards. 
  • Option C: The traditional definition of a family has changed and may include people who may not live in the same house as the client. Many people consider friends to be as close or even closer than extended (or immediate) family. People who have lost close family members or have become removed from them may create a family unit of friends with similar interests and goals to become replacements or enhancements to a lacking family structure.
  • Option D: Family members are defined by the client, not by the nurse. Who comprises a family is up to the people in the family themselves. People may opt to keep blood relatives in their lives, or let them go if they are toxic to their well-being. Many folks add caring and supportive people to their extended clan when they choose, deciding who belongs in their specific definition of family.
  • Option E: In addition to a universal family definition, plenty of people consider a group of friends to be family, and many consider pets as defining members of the family unit.
  • Option F: Family members may also include those people who provide for the physical and emotional needs of the client. The traditional definition of a family has changed and may include people not related by blood or marriage, those of a different racial background, and those who may not live in the same house as the client. 

NURSESLABS-SATA-1-026

The nurse recognizes that a client is experiencing insomnia when the client reports which of the following? Select all that apply.

  • A. Extended time to fall asleep
  • B. Falling asleep at inappropriate times
  • C. Difficulty staying asleep
  • D. Feeling tired after a night’s sleep

Correct Answer: A, C, & D.

These symptoms are often reported by clients with insomnia. Clients report nonrestorative sleep. Arising once at night to urinate (nocturia) is not in and of itself insomnia. According to the third edition of the International Classification of Sleep Disorders (ICSD-3), insomnia is characterized by difficulty in either initiating sleep, maintaining sleep continuity, or poor sleep quality.

  • Option A: Sleep-onset insomnia refers to difficulty falling asleep. This type of insomnia may occur with people who have a hard time relaxing in bed, as well as people whose circadian rhythm is not in sync due to factors like jet lag or irregular work schedules.
  • Option B: Hypersomnia is generally seen in adolescents or young adults. The patients with hypersomnia complain of disabling excessive daytime sleepiness. They find it difficult to maintain alertness during the major waking hours of the day with sleep occurring unintentionally or at inappropriate times that interfere with the daily routine.
  • Option C: Sleep maintenance insomnia refers to difficulty staying asleep after initially nodding off. This type of insomnia is common in elderly sleepers, as well as people who consume alcohol, caffeine, or tobacco before bed. Certain disorders like sleep apnea and periodic limb movement disorder can also cause sleep maintenance insomnia.
  • Option D: Chronic insomnia can adversely affect the health, quality of life, academic performance, increase the risk of motor vehicle accidents, decrease the productivity at work, irritability and increase daytime sleepiness.

NURSESLABS-SATA-1-027

The nurse teaches the mother of a newborn that in order to prevent sudden infant death syndrome (SIDS) the best position to place the baby after nursing is? Select all that apply.

  • A. Prone
  • B. Side-lying
  • C. Supine
  • D. Fowler’s

Correct Answer: B & C.

Research demonstrates that the occurrence of SIDS is reduced with these two positions. Sudden infant death syndrome (SIDS) is the abrupt and unexplained death of an infant less than 1-year old. Despite a thorough investigation (a careful review of clinical history, death scene investigation, and a complete autopsy), a cause for the patient’s demise is not identified.

  • Option A: Studies suggest that the prone positioning predisposes to suffocation, resulting from decreased arousal, the type of bedding material, and overheating. Studies have demonstrated that prone sleeping is associated with longer sleep duration, longer obstructive events, and decreased arousal.
  • Option B: Side sleeping can also be safe as the baby grows and gets stronger. The baby gets more and more active during sleep as they approach their first birthday — which, thankfully, is also when a lot of these sleep-position worries go away.
  • Option C: The incidence of SIDS declined by more than 50 percent in the United States after physicians began to promote “On the back to sleep.” After the American Academy of Pediatrics (AAP) issued a recommendation for supine sleeping in 1992, the incidence of SIDS decreased.
  • Option D: Back sleeping with an alternating head position is best. It’s true that babies are born with softer skulls. They also have weak neck muscles in the early months of life. Give the baby plenty of supervised tummy time during the day. This helps to prevent a flat head and encourages the baby to develop their neck, arm, and upper-body muscles.

NURSESLABS-SATA-1-028

A client has a diagnosis of primary insomnia. Before assessing this client, the nurse recalls the numerous causes of this disorder. Select all that apply.

  • A. Chronic stress
  • B. Severe anxiety
  • C. Generalized pain
  • D. Excessive caffeine
  • E. Chronic depression
  • F. Environmental noise

Correct Answer: A, D, & F.

Acute or primary insomnia is caused by emotional or physical discomfort not caused by the direct physiologic effects of a substance or a medical condition. 

  • Option A: This type of insomnia is usually idiopathic, although it can be impacted by mild to moderate stress. Idiopathic insomnia is truly without any identifiable contributory factor, while stress-related insomnia can be characterized by mild stress, such as rumination or other thoughts throughout the night.
  • Option B: Primary idiopathic insomnia occurs without any identifiable cause and in the absence of anxiety. Developmental issues during childhood, for example, separation anxiety, may predispose a child to develop sleep problems. People with certain personality traits like perfectionism, ambitiousness, neuroticism, low extraversion, and susceptibility to depression and worry are more likely to develop insomnia over time.
  • Option C: Comorbid medical issues like restless legs syndrome, chronic pain, gastroesophageal reflux disease (GERD), respiratory issues, and immobility are associated with risk of chronic insomnia.
  • Option D: Excessive caffeine intake is an example of disruptive sleep hygiene; caffeine is a stimulant that inhibits sleep. Coffee, tea, cola and other caffeinated drinks are stimulants. Drinking them in the late afternoon or evening can keep the client from falling asleep at night.
  • Option E: The sleep problems of primary insomnia are not associated with lifestyle habits or a medical or psychiatric cause. Individuals who have difficulty coping with a stressful situation or those who report being habitual light sleepers have an elevated propensity to develop chronic insomnia. There is a high rate of association between insomnia and psychiatric disorders like depression, anxiety, and post-traumatic stress disorder.
  • Option F: Environmental noise causes physical and/or emotional effects and therefore is related to primary insomnia. Poor sleep habits include an irregular bedtime schedule, naps, stimulating activities before bed, an uncomfortable sleep environment, and using the bed for work, eating or watching TV.

NURSESLABS-SATA-1-029

The use of barbiturates in treating insomnia include which of the following? Select all that apply.

  • A. Barbiturates deprive people of NREM sleep
  • B. Barbiturates deprive people of REM sleep
  • C. When the barbiturates are discontinued, the NREM sleep increases.
  • D. When the barbiturates are discontinued, the REM sleep increases.
  • E. Nightmares are often an adverse effect when discontinuing barbiturates.

Correct Answer: B, D, & E.

Barbiturates are a group of sedative-hypnotic medications used for the treatment of seizure disorder, neonatal withdrawal, insomnia, preoperative anxiety, induction of coma for increased intracranial pressure

  • Option A: The demonstration of a relationship between the profusion of eye movements and the “activity” or vividness of the accompanying dream and the finding that barbiturates not only decrease the overall amount of REM sleep but also reduce the profusion of eye movements per minute of REM sleep led to the prediction that barbiturate administration would result in dream experiences of a more tranquil nature
  • Option B: Barbiturates deprive people of REM sleep. To determine the effect of barbiturates on sleep, two subjects, after a control period, received 200 mg. of sodium amylobarbitone for 26 nights. All night sleep records taken during this period showed that the barbiturate shortened the delay to sleep, increased the total sleep period, lengthened the delay to rapid eye movement (R.E.M.) sleep, and depressed R.E.M. sleep.
  • Option C: After five nights R.E.M. sleep returned to baseline values —that is, showed tolerance. On stopping the drug withdrawal phenomena were seen, even to this small dose of the drug. In a second experiment a subject dependent on 600 mg. of Tuinal was found to have low normal R.E.M. sleep while on drugs. On withdrawal, delay to sleep increased and total sleep time fell. R.E.M. sleep was doubled and the delay to R.E.M. became abnormally short.
  • Option D: When the barbiturate is stopped and REM sleep once again occurs, a rebound phenomenon occurs. During this phenomenon, the person’s dream time constitutes a larger percentage of the total sleep pattern, and the dreams are often nightmares.

NURSESLABS-SATA-1-030

Which of the following is appropriate when there is a benzodiazepine overdose? Select all that apply.

  • A. Administration of syrup of ipecac
  • B. Gastric lavage
  • C. Activated charcoal and a saline cathartic
  • D. Hemodialysis
  • E. Administration of Flumazenil

Correct Answer: B, C, & E.

Benzodiazepines are currently used to treat anxiety, seizures, withdrawal states, insomnia, agitation, and are commonly used for procedural sedation. Due to their many uses and addictive properties, benzodiazepines have been widely prescribed and abused since their development several decades ago.

  • Option A: The administration of syrup of ipecac is contraindicated because of aspiration risks related to sedation. Ipecac, or syrup of ipecac (SOI), is a medication once used to induce vomiting. Its medical use has virtually vanished, and it is no longer recommended for routine use in toxic ingestion. The abuse of SOI as a purgative in eating disorders, however, is increasing.
  • Option B: If ingestion is recent, the decontamination of the GI system is indicated. Gastric lavage is generally the best and most effective means of gastric decontamination. Occasionally, gastric lavage and administration of activated charcoal is indicated, but only if the patient is awake and potentially sensitive to benzodiazepines and if a large dose has been ingested within the last 1 to 2 hours.
  • Option C: Activated charcoal and a saline cathartic may be administered to remove any remaining drug. Early administration of activated charcoal in patients able to protect their airway is only needed if there are coingestants.
  • Option D: Hemodialysis is not useful in the treatment of benzodiazepine overdose. Forced diuresis and dialysis techniques are not indicated since they will not significantly accelerate the elimination of these agents.
  • Option E: Flumazenil is a nonspecific competitive antagonist at the benzodiazepine receptor that can reverse benzodiazepine-induced sedation. Flumazenil can be used to acutely reverse the sedative effects of benzodiazepines, though this is normally done only in cases of extreme overdose or sedation. 

NURSESLABS-SATA-2-001

A 6-year-old child with leukemia is hospitalized and is receiving combination chemotherapy. Laboratory results indicate that the child is neutropenic, and the nurse prepares to implement protective isolation procedures. Which interventions would the nurse initiate? Select all that apply.

  • A. Restrict all visitors.
  • B. Place the child on a low-bacteria diet.
  • C. Change dressings using sterile technique.
  • D. Encourage the consumption of fresh fruits and vegetables.
  • E. Perform meticulous hand washing before caring for the child.
  • F. Allow fresh-cut flowers in the room as long as they are kept in a vase with fresh water.

Correct Answer: B, C, & E.

Leukemias are a group of hematologic disorders characterized by the dysfunctional proliferation and development of leukocytes. Many genetic and environmental risk factors have been identified, though the exact cause of most leukemia subtypes is unknown.

  • Option A: Not all visitors need to be restricted, but anyone who is ill should not be allowed in the child’s room. Bone marrow suppression, neutropenia, and chemotherapy places the patient at high risk for infection.
  • Option B: The child is placed on a low-bacteria diet. Provide a nutritious diet, high in protein and calories, avoiding raw fruits, vegetables, or uncooked meats. Proper nutrition enhances the immune system. Minimizes potential sources of bacterial contamination.
  • Option C: Dressings are always changed with sterile technique. Provide thorough skin care by keeping the patient’s skin and perianal area clean, apply mild lotion or creams to keep the skin from drying or cracking. Thoroughly clean skin before all invasive skin procedures.
  • Option D: Fruits and vegetables not peeled before being eaten harbor molds and should be avoided until the white blood cell count rises. Restrict fresh fruits and make sure they are properly washed or peeled.
  • Option E: Meticulous hand washing is required before caring for the child. In addition, gloves, a mask, and a gown are worn (per agency policy). This prevents cross-contamination and reduces risk of infection.
  • Option F: For the hospitalized neutropenic child, flowers or plants should not be kept in the room because standing water and damp soil harbor Aspergillus and Pseudomonas, to which these children are very susceptible.

NURSESLABS-SATA-2-002

A 16-year-old child is brought to the emergency department by his mother with a complaint that the child just experienced a tonic-clonic seizure. On arrival in the emergency department, no apparent seizures were occurring. The mother states that her son is taking medication for the seizure disorder. The nurse plans care, knowing that which of the following medications are used for long-term control of tonic-clonic seizures? Select all that apply.

  • A. Diazepam (Valium)
  • B. Alprazolam (Xanax)
  • C. Gabapentin (Neurontin)
  • D. Ethosuximide (Zarontin)
  • E. Carbamazepine (Tegretol)
  • F. Methylphenidate (Ritalin)

Correct Answer: C, D, & E.

Medications that are prescribed for long-term control of tonic-clonic seizures are gabapentin, ethosuximide, and carbamazepine. The decision to start chronic, prophylactic antiseizure medications is individualized based on numerous factors, including the chance of event being a seizure, confirmation of seizure based on history and physical examination, patient stability, and risk of recurrent seizures.

  • Option A: Diazepam is a medication that is prescribed to halt tonic-clonic episodes. Diazepam is a benzodiazepine medication that is FDA approved for the management of anxiety disorders, short-term relief of anxiety symptoms, spasticity associated with upper motor neuron disorders, adjunct therapy for muscle spasms, preoperative anxiety relief, management of certain refractory epilepsy patients, and adjunct in severe recurrent convulsive seizures, and an adjunct in status epilepticus.
  • Option B: Alprazolam is a medication used to treat anxiety. Alprazolam is frequently prescribed to manage panic and anxiety disorders. Alprazolam has also been misused for recreational purposes because of its disinhibition, euphoria, and anxiolytic effects. 
  • Option C: Gabapentin is an anticonvulsive medication that originally saw use as a muscle relaxer and anti-spasmodic medication, but later it was discovered it had the potential of the medication as anticonvulsant medication and as an adjunct to more potent anticonvulsants.
  • Option D: Ethosuximide is FDA approved for the management of absence seizures in patients over 3 years of age. Currently, there are no off-label uses for ethosuximide; however, there is some evidence it may have some analgesic effects.
  • Option E: Carbamazepine is used to manage and treat epilepsy, trigeminal neuralgia, and acute manic and mixed episodes in bipolar I disorder. Indications for epilepsy are specifically for partial seizures with complex symptomatology (psychomotor, temporal lobe), generalized tonic seizures (grand mal), and mixed seizure patterns.
  • Option F:  Methylphenidate is a medication used to treat attention deficit hyperactivity disorder. These medications are not suitable for long-term control of a seizure condition. Children diagnosed with ADHD should be at least six years of age or older before being started on this medication.

NURSESLABS-SATA-2-003

A child has been diagnosed with meningococcal meningitis. Which of the following isolation techniques is appropriate? Select all that apply.

  • A. Enteric precautions
  • B. Neutropenic precautions
  • C. No precautions are required as long as antibiotics have been started
  • D. Isolation precautions for at least 24 hours after the initiation of antibiotics
  • E. Droplet precautions (private room, mask for all entering the room) until they have completed 24 hours of appropriate antibiotic therapy
  • F. Negative pressure ventilation is not required.

Correct Answer: D, E, & F.

Meningococcal meningitis is the term used to describe a bacterial form of meningitis caused by Neisseria meningitidis. This form of meningitis is associated with high morbidity and mortality. Meningococcal meningitis is a medical emergency for which symptoms can range from transient fever to fulminant bacteremia and septic shock.

  • Option A: Enteric precautions are taken to prevent infections that are transmitted primarily by direct or indirect contact with fecal material. They’re indicated for patients with known or suspected infectious diarrhea or gastroenteritis. Clostridium difficile is the most common cause of hospital-acquired infectious diarrhea.
  • Option B: Neutropenic precautions are steps to take to prevent infections if there is moderate to severe neutropenia. Neutropenia is a condition that causes low neutrophils in the blood.
  • Option C: Patients with pneumococcal or viral meningitis do not require isolation. Viruses can penetrate the central nervous system (CNS) via retrograde transmission along neuronal pathways or by hematogenous seeding.
  • Option D: Isolation is begun and maintained for at least 24 hours after antibiotics are given. On initial encounter when the patient presents with an undifferentiated acute bacterial meningitis, administration of broad-spectrum antibiotics is appropriate pending bacterial isolation. Seven days course of antibiotic therapy is usually sufficient to treat suspected cases of meningococcal meningitis.
  • Option E: Meningococcal meningitis is transmitted primarily by droplet infection. The patient with suspected or confirmed N. meningitidis should follow droplet precaution. This should be continued until after 24 hours of effective antibiotics administration.
  • Option F: Negative pressure ventilation is not required. Droplet Precautions are intended to prevent transmission of pathogens spread through close respiratory or mucous membrane contact with respiratory secretions as described. Because these pathogens do not remain infectious over long distances in a healthcare facility, special air handling and ventilation are not required to prevent droplet transmission.

NURSESLABS-SATA-2-004

A client enters the emergency department confused, twitching, and having seizures. His family states he recently was placed on corticosteroids for arthritis and was feeling better and exercising daily. On data collection, he has flushed skin, dry mucous membranes, an elevated temperature, and poor skin turgor. His serum sodium level is 172 mEq/L. Choose the interventions that the health care provider would likely prescribe. Select all that apply.

  • A. Monitor intake and output.
  • B. Monitor vital signs.
  • C. Maintain a sodium-reduced diet.
  • D. Monitor electrolyte levels.
  • E. Increase water intake orally.
  • F. Administer sodium replacements.

Correct Answer: A, B, C, D, and E.

Hypernatremia is described as having a serum sodium level that exceeds 145 mEq/L. Signs and symptoms would include dry mucous membranes, loss of skin turgor, thirst, flushed skin, elevated temperature, oliguria, muscle twitching, fatigue, confusion, and seizures. Interventions include monitoring fluid balance, monitoring vital signs, reducing dietary intake of sodium, monitoring electrolyte levels, and increasing oral intake of water. 

  • Option A: Monitor intake and output and specific gravity. Assess the presence and location of edema. Weigh the client daily. These parameters are variable, depending on the fluid status, and are indicators of therapy needs and effectiveness.
  • Option B: Depending on the fluid status, hypertension or hypotension may be present. The presence of postural hypotension may affect activity tolerance. Metabolic acidosis secondary to hyperchloremia may result in deep, labored breathing with air hunger, which can lead to a cardiopulmonary arrest if left untreated.
  • Option C: Teach the client to avoid foods high in sodium such as regular canned vegetables and vegetable juices, processed foods, snack foods, and condiments. Decreases the risk of sodium associated complications such as stroke, heart disease, and heart failure.
  • Option D: Monitor serum electrolytes, osmolality, and arterial blood gasses, as indicated. This will evaluate the therapy needs and effectiveness.
  • Option E: Encourage increased oral and IV fluid intake. Replacement of total body water deficit will gradually restore sodium and water balance.
  • Option F: Sodium replacement therapy would not be prescribed for a client with hypernatremia. Sodium intake restriction while promoting renal clearance decreases serum sodium levels in the presence of extracellular fluid excess.

NURSESLABS-SATA-2-005

A client has died, and a nurse asks a family member about the funeral arrangements. The family member refuses to discuss the issue. The nurse’s appropriate action is to? Select all that apply.

  • A. Show acceptance of feelings.
  • B. Provide information needed for decision making.
  • C. Suggest a referral to a mental health professional.
  • D. Remain with the family member without discussing funeral arrangements.
  • E. Let the family slowly acknowledge its impact.

Correct Answer: D & E.

Grief is a process that can begin long before the loss of a loved one. Similar to the stages of dying, individuals go through a process to help them eventually cope and be able to live with that loss. People never get over their loss, but find ways to live with the loss and without their deceased loved one (ELNEC, 2010).

  • Option A: This is an appropriate intervention for the acceptance or reorganization and restitution stage. In this final stage of grief, the person accepts the reality of the loss. It can’t be reversed. Although he or she still feels sad, he or she is ready to start moving on in life.
  • Option B: This may be an appropriate intervention for the bargaining stage. During this stage, he or she dwells on what could’ve done to counteract the loss. General thoughts are “If only…” and “What if…”.
  • Option C: This may be an appropriate intervention for depression. Sadness sets in as the person begins to understand the loss and its effect in life. Indications of depression include crying, sleep issues, and a decreased appetite.
  • Option D: The family member is exhibiting the first stage of grief (denial), and the nurse should remain with the family member. One of the biggest facilitators of this process which nurses can engage in is active listening. By actively listening to the bereaved, it helps them express their feelings and feel as though they are being heard.
  • Option E: As the family moves through the experience and slowly acknowledges its impact, the initial denial and disbelief fade. Bereavement includes grief and mourning and has been considered to be the “time period in which the survivor adjusts to their life without their loved one” (ELNEC, 2010). This period can include the time right after the loss or death occurs, during the funeral proceedings, and during the grieving process afterward.

NURSESLABS-SATA-2-006

A client is scheduled for a myelogram, and the nurse provides a list of instructions to the client regarding preparation for the procedure. Which instructions should the nurse place on the list? Select all that apply.

  • A. Jewelry will need to be removed.
  • B. An informed consent will need to be signed.
  • C. A trained x-ray technician performs the procedure.
  • D. The procedure will take approximately 45 minutes.
  • E. A liquid diet can be consumed on the day of the procedure.
  • F. Solid food intake needs to be restricted only on the day of the procedure.

Correct Answer: A, B, & D.

A myelogram is an X-ray exam in which a contrast agent (X-ray dye) is injected into the spinal canal to visualize the bones, discs, muscles and nerves. A myelogram is used to detect abnormalities of the spine such as disc problems, tumors and bone spurs, narrowing of the spinal canal or malformations of the spine.

  • Option A: The client will need to remove jewelry and metal objects from the chest area. Try to wear non-restrictive, comfortable clothing and slip on shoes if possible. Remove all piercings and leave all jewelry and valuables at home.
  • Option B: An informed consent is required because the procedure is invasive. A myelogram may be done to assess the spinal cord, subarachnoid space, or other structures for changes or abnormalities.
  • Option C: The procedure is performed by the healthcare provider. The technologist will verify identification and exam requests. The technologist and radiologist will be available to answer any questions.
  • Option D: The client is told that the procedure takes about 45 minutes. The patient will lie on the stomach on the X-ray table. If the patient cannot tolerate lying on the stomach for at least 30 minutes, notify the doctor.
  • Option E: If not on a fluid restriction, drink at least 6-8 glasses of fluid the day before the procedure. Do not eat or drink anything for 4 hours before the exam.
  • Option F: Client preparation for a myelogram includes instructing the client to restrict food and fluids for 4 to 8 hours before the procedure. The client is also told that pretest medications may be prescribed for relaxation.

NURSESLABS-SATA-2-007

A client with a closed head injury is receiving phenytoin (Dilantin), an anticonvulsant medication. Which of the following would indicate that the client is experiencing side effects related to this medication? Select all that apply.

  • A. Ataxia
  • B. Sedation
  • C. Constipation
  • D. Bleeding gums
  • E. Hyperglycemia
  • F. Decreased platelet count

Correct Answer: C, D, E, & F.

Phenytoin toxicity can occur from an increase in the daily dose of phenytoin, changes in the formulations or brands as well as changes in the frequency of administration. It can also occur when patients are started on new medications that interact with the metabolism or binding capacity of phenytoin to plasma proteins.

  • Option A: Ataxia is a side effect of benzodiazepines. In most patients, symptoms occur within days or weeks after the introduction of a new drug or an increase in dose. In general, ataxia tends to disappear after discontinuation of the drug, but chronic ataxia has been described for some drugs.
  • Option B: Sedation is a side effect of barbiturates, not phenytoin. Unlike benzodiazepines and other anticonvulsants, phenytoin does not cause sedation in therapeutic doses. Phenytoin can be given orally and intravenously. Therapeutic levels are 10 to 20 mg/L. Adverse effects associated with phenytoin use can be subdivided based on acute use versus chronic use.
  • Option C: It contributes to constipation as well. In therapeutic doses, phenytoin is absorbed entirely and reaches peak plasma concentration at 1.5 to 3 hours. However, in settings of acute ingestions, absorption tends to last longer than two weeks; this is potentially attributable to its effects on reducing the gastrointestinal motility and poor water solubility.
  • Option D: Gingival hyperplasia can occur, causing gums to bleed easily. The etiology of phenytoin-induced gingival enlargement (PIGE) is likely due to the direct effects of the drug and its metabolites on the gingival fibroblasts.
  • Option E: Blood glucose levels can elevate when taking phenytoin. Phenytoin-induced hyperglycemia may be a serious problem in nondiabetic as well as diabetic patients. Non- ketotic hyperosmolar coma and death have also been associated with use of the drug. Phenytoin should be ad- ministered with caution, particularly to patients with an existing error in glucose metabolism.
  • Option F: Dilantin causes blood dyscrasias, such as decreased platelet counts and decreased white blood cell counts. Phenytoin has been reported to induce various hematologic reactions, including thrombocytopenia. An intermediate epoxide metabolite of phenytoin is suspected as the cause of platelet destruction, which may occur via a complement-antibody reaction.

NURSESLABS-SATA-2-008

A client with carcinoma of the lung develops the syndrome of inappropriate antidiuretic hormone (SIADH) as a complication of cancer. The nurse anticipates that which of the following may be prescribed? Select all that apply.

  • A. Radiation
  • B. Chemotherapy
  • C. Increased fluid intake
  • D. Serum sodium blood levels
  • E. Decreased oral sodium intake
  • F. Medication that is antagonistic to antidiuretic hormone (ADH)

Correct Answer: A, B, D, & F.

Cancer is a common cause of SIADH. In clients with SIADH, excessive amounts of water are reabsorbed by the kidney and put into the systemic circulation. The increased water causes hyponatremia (decreased serum sodium levels) and some degree of fluid retention. 

  • Option A: Syndrome of inappropriate antidiuretic hormone (SIADH) has been commonly associated with small cell carcinoma and is often seen in these patients. However, SIADH associated with squamous cell carcinoma has rarely been reported on, and the mechanism for this rare association is still unknown.
  • Option B: The immediate institution of appropriate cancer therapy (usually either radiation or chemotherapy) can cause tumor regression so that ADH synthesis and release processes return to normal.
  • Option C: Hyponatremia treatment needs to be personalized based on severity and duration of sodium serum reduction, extracellular fluid volume and etiology. However, literature data highlight the importance of early correction of the serum concentration levels. To achieve this the main options are fluid restriction, hypertonic saline, loop diuretics, isotonic saline, tolvaptan and urea. 
  • Option D: Sodium levels are monitored closely because hypernatremia can suddenly develop as a result of treatment. Firstly, it is recommended to detect the cause of reduced sodium concentration, although the increase in sodium concentration levels is likely to be the main issue in life-threatening hyponatremia.
  • Option E: SIADH is managed by treating the condition and its cause, and treatment usually includes fluid restriction, increased sodium intake, and a medication with a mechanism of action that is antagonistic to ADH. 
  • Option F: For patients affected by SIADH, vaptans represent a new class of drug antagonizing the V2 receptor on renal tubular cells. To date, two molecules are approved: conivaptan and tolvaptan. The prescription of conivaptan (intravenous use) has been authorized for euvolemic hyponatremia due to SIADH by the United States (US) Food and Drug Administration (FDA) but not by the European Medicines Agency (EMA).

NURSESLABS-SATA-2-009

The nurse is preparing to teach a client about the prescribed spironolactone (Aldactone) to monitor for adverse effects of the drug. The nurse should instruct the client about which adverse effects? Select all that apply.

  • A. Confusion.
  • B. Fatigue.
  • C. Hypertension.
  • D. Leg cramps.
  • E. Weakness.
  • F. Urinary retention.

Correct Answer: A, B, & E.

Spironolactone (Aldactone) is used to treat hypertension and edema by removing excess fluid. Aldactone is known as a potassium-sparing diuretic. Confusion, fatigue, and weakness are signs of hyperkalemia, an adverse effect of spironolactone. 

  • Option A: One study mentions the following additional adverse effects in order from more to less common: dehydration, hyponatremia, gastrointestinal problems (nausea, vomiting, diarrhea or anorexia), neurological abnormalities (headache, drowsiness, asterixis, confusion, or coma), and skin rashes.
  • Option B: Spironolactone blocks the hormone aldosterone, which can lead to fatigue. In addition, it can lower the blood pressure, and if this drop is sudden, the client may feel tired.
  • Option C: Spironolactone is used to treat hypertension, so it would not produce this effect. Spironolactone is recommended in patients with resistant hypertension which is defined as uncontrolled blood pressure despite three antihypertensive drug combinations including a diuretic. Spironolactone is a mineralocorticoid receptor antagonist and causes anti-androgenic side effects.
  • Option D: Leg cramps are an adverse effect of hypokalemia. Hyperkalemia is an adverse effect of spironolactone. This drug is contraindicated in patients with hyperkalemia and in those at increased risk of developing hyperkalemia.
  • Option E: Symptoms of hypokalemia may include attacks of severe muscle weakness, eventually leading to paralysis and possibly respiratory failure. Muscular malfunction may result in paralysis of the bowel, low blood pressure, muscle twitches and mineral deficiencies (tetany).
  • Option F: Urinary retention is a side effect of anticholinergics. Medications with anticholinergic properties, such as tricyclic antidepressants, cause urinary retention by decreasing bladder detrusor muscle contraction.

NURSESLABS-SATA-2-010

The clinic nurse is assisting to perform a focused data collection process on a client who is complaining of symptoms of a cold, a cough, and lung congestion. Which of the following would the nurse include for this type of data collection? Select all that apply.

Questions and rationale from Nurseslabs.com Feel free to print or share and link back to us! For more practice questions, please visit our Nursing Test Bank [https://nurseslabs.com/nursing-test-bank]

  • A. Auscultating lung sounds
  • B. Obtaining the client’s temperature
  • C. Checking the strength of peripheral pulses
  • D. Obtaining information about the client’s respirations
  • E. Performing a musculoskeletal and neurological examination
  • F. Asking the client about a family history of any illness or disease

Correct Answer: A, B, & D.

A focused data collection process focuses on a limited or short-term problem, such as the client’s complaint. Because the client is complaining of symptoms of a cold, a cough, and lung congestion the nurse would focus on the respiratory system and the presence of an infection. 

  • Option A: Auscultation of the lungs should be systematic and follow a stepwise approach in which the examiner surveys all the lung zones. For practical purposes, the lung can be divided into apical, middle and basilar regions during auscultation.
  • Option B: An increase in temperature may be a sign of underlying infection. The diagnosis of a cough is an obvious clinical observation. A cough is a symptom rather than a diagnosis of disease. As such, many patients present for evaluation of the secondary or underlying effects of cough rather than a cough itself.
  • Option C: Checking the strength of peripheral pulses relates to a vascular assessment, which is not related to this client’s complaints. Otherwise, a systemic approach should be used to identify any coexisting illness, which may be the origin or compounding factor of a cough.
  • Option D: During the inspection, the examiner should pay attention to the pattern of breathing: thoracic breathing, thoracoabdominal breathing, costal markings, and use of accessory breathing muscles. The use of accessory breathing muscles (i.e., scalenes, sternocleidomastoid muscle, intercostal muscles) could point to excessive breathing effort caused by pathologies.
  • Option E: A musculoskeletal and neurological examination also is not related to this client’s complaints. However, the strength of peripheral pulses and a musculoskeletal and neurological examination would be included in a complete data collection.
  • Option F: A complete data collection includes a complete health history and physical examination and forms a baseline database. Likewise, asking the client about a family history of any illness or disease would be included in a complete assessment.

NURSESLABS-SATA-2-011

A community health nurse is conducting a teaching session about terrorism with members of the community and discussing information regarding anthrax. The nurse tells those attending that anthrax can be transmitted via which route(s)? Select all that apply.

  • A. Skin
  • B. Kissing
  • C. Inhalation
  • D. Gastrointestinal
  • E. Direct contact with an infected individual
  • F. Sexual contact with an infected individual

Correct Answer: A, C, & D.

Anthrax is caused by Bacillus anthracis, and it can be contracted through the digestive system, abrasions in the skin, or inhalation. It cannot be spread from person to person.

  • Option A: Skin contact results in cutaneous anthrax. Cutaneous anthrax results from inoculation of B. anthracis spores through the abraded skin into subcutaneous tissues. The bacteria subsequently germinate and multiply locally and begin toxin production.  This leads to the characteristic edema and cutaneous ulceration. 
  • Option B: Viruses responsible for diseases such as hepatitis viruses, herpesvirus infections (e.g., with Herpes simplex types 1 and 2, Epstein-Barr virus, Cytomegalovirus, and Kaposi syndrome herpesvirus), and papillomaviruses can be conveyed by kissing—as can potentially other viruses present in saliva such as Ebola and Zika viruses.
  • Option C: Inhalation or ingestion of the spores leads to inhalational or gastrointestinal (GI) anthrax. Inhalational anthrax leads to accumulation of B. anthracis spores within the lung alveoli. The spores are engulfed by immune cells (macrophages, neutrophils, dendritic cells) and transported to regional lymph nodes where the bacteria germinate, multiply, and begin toxin production.
  • Option D: Human transmission occurs via contact with infected animals through butchering and working with hides or ingestion of raw or undercooked meat. GI anthrax occurs due to ingestion of contaminated meat, with spores introduced into the gastrointestinal tract, causing bacterial replication, mucosal ulcerations, and bleeding. 
  • Option E: Anthrax is acquired from animals; there are no reports of direct human to human transmission.
  • Option F: More than 30 different bacteria, viruses and parasites are known to be transmitted through sexual contact. Eight of these pathogens are linked to the greatest incidence of sexually transmitted disease. Of these 8 infections, 4 are currently curable: syphilis, gonorrhoea, chlamydia and trichomoniasis. The other 4 are viral infections which are incurable: hepatitis B, herpes simplex virus (HSV or herpes), HIV, and human papillomavirus (HPV).

NURSESLABS-SATA-2-012

The emergency room nurse is providing discharge teaching to the parents of a 2-year-old child who sustained burns from a hot cup of coffee that had been left on the kitchen counter. The nurse evaluates that the parents have correctly understood the teaching when they state which of the following? Select all that apply.

  • A. “We will be sure to not leave hot liquids unattended.”
  • B. “I guess my child needs to understand what the word ‘hot’ means.”
  • C. “We will be sure that our child stays in his room when we work in the kitchen.”
  • D. “We will install a safety gate as soon as we get home so that our child can’t get into the kitchen.”
  • E. “We will not put adhesive bandages over the affected area.”

Correct Answer: A & E.

Toddlers, with their increased mobility and developing motor skills, can reach hot water, open fires, or hot objects placed on counters and stoves above their eye level. Pot handles should be turned inward and toward the middle of the stove. Options 2, 3, and 4 do not reflect an adequate understanding of the principles of safety.

  • Option A: Hot liquids should never be left unattended, and the toddler should always be supervised. Don’t cook, drink, or carry hot beverages or foods while holding a child. Keep hot foods and liquids away from table and counter edges. Don’t use tablecloths or place mats, which young children can pull down.
  • Option B: Store items designed to get hot, such as clothes irons or curling irons, unplugged and out of reach. Be careful with food or liquids warmed in a microwave, which might heat foods unevenly.
  • Option C: Don’t leave the stove unattended when cooking. Parents should be encouraged to remain in the kitchen when preparing a meal and reminded to use the back burners on the stove.
  • Option D: Block access to the stove, fireplace, space heaters and radiators. Don’t leave a child unattended in a room when these items are in use.
  • Option E: Parents should not put adhesive bandages on very young kids, though, as these can be a choking hazard if they get loose. 

NURSESLABS-SATA-2-013

A licensed practical nurse is attending an agency orientation meeting about the nursing model of practice implemented in the facility. The nurse is told that the nursing model is a team nursing approach. The nurse understands that which of the following is a characteristic of this type of nursing model of practice? Select all that apply.

  • A. A task approach method is used to provide care to clients.
  • B. Managed care concepts and tools are used when providing client care.
  • C. Nursing staff are led by a nurse when providing care to a group of clients.
  • D. A single registered nurse is responsible for providing nursing care to a group of clients.
  • E. This model utilizes the diversity of skills, education, and qualification level of the entire staff.

Correct Answer: C & E.

The team nursing approach allows our nurses to move from caring about “my” patients to teams caring for a group of “our patients.”  Team Nursing reduces the stress of one RN trying to care for a group of assigned patients by themselves, with the goal of each member of the team sharing the workload.

  • Option A: This identifies functional nursing. Functional nursing revolves around team ethic, and it produces a catalog of processes, which are undertaken by different individuals to ensure efficient patient care delivery.
  • Option B: This identifies a component of case management. Case management is a collaborative process of assessment, planning, facilitation, care coordination, evaluation and advocacy for options and services to meet an individual’s and family’s comprehensive health needs.
  • Option C: The team nursing model of care involves pairing nurses who work as a team to deliver patient care. In team nursing, nursing personnel are led by a nurse when providing care to a group of clients. 
  • Option D: This identifies primary nursing. This model favors a more holistic approach to patient care, as it involves a single senior nurse taking responsibility for the patient throughout their hospital stay.
  • Option E: This model utilizes the diversity of skills, education, and qualification level of the entire staff. Team members work collaboratively and share responsibility. In the team nursing model, an experienced nurse for the unit or floor oversees the work of a team of clinicians and support staff for a group of patients.

NURSESLABS-SATA-2-014

A licensed practical nurse is planning the client assignments for the day. Which of the following is the most appropriate assignment for the nursing assistant? Select all that apply.

  • A. A client who requires wound irrigation
  • B. A client who requires frequent ambulation
  • C. A client who is receiving continuous tube feedings
  • D. A client who requires frequent vital signs after a cardiac catheterization
  • E. A client who needs to be turned or repositioned in bed

Correct Answer: B and E.

The nurse must determine the most appropriate assignment on the basis of the skills of the staff member and the needs of the client.

  • Option A: Wound irrigations and tube feedings are not performed by unlicensed personnel. The staff members’ levels of education, knowledge, past experiences, skills, abilities, and competencies are also evaluated and matched with the needs of all of the patients in the group of patients that will be cared for.
  • Option B: In general, simple, routine tasks such as making unoccupied beds, supervising patient ambulation, assisting with hygiene, and feeding meals can be delegated. But if the patient is morbidly obese, recovering from surgery, or frail, work closely with the UAP or perform the care yourself.
  • Option C: Care of the client receiving continuous tube feedings should be delegated to another registered nurse because it requires monitoring. Scopes of practice are also considered prior to the assignment of care. All states have scopes of practice for advanced nurse practitioners, registered nurses, licensed practical nurses and unlicensed assistive personnel like nursing assistants and patient care technicians.
  • Option D: The client who had a cardiac catheterization will require specific monitoring in addition to that of the vital signs. Based on the basic entry educational preparation differences among these members of the nursing team, care should be assigned according to the level of education of the particular team member.
  • Option E: In this case, the most appropriate assignment for a nursing assistant would be to care for the client who requires client repositioning. The nursing assistant is skilled in these tasks.

NURSESLABS-SATA-2-015

A male client who has heart failure receives an additional dose of bumetanide as prescribed 4 hours after the daily dose. The nurse assesses him 15 minutes after administering the medication and reminds him to save all urine in the bathroom. Thirty minutes later the nurse finds the client on the floor, unresponsive, and bleeding from a laceration. Determine the issues that support the client’s malpractice claim. Select all that apply.

  • A. Failure to replace body fluids
  • B. Increased risk of hypotension
  • C. Failure to teach the client adequately
  • D. Increased need to protect the client
  • E. Excessive bumetanide administration
  • F. Lack of follow-up nursing actions

Correct Answer: B, C, D, & F.

To prove malpractice against a nurse, the plaintiff must prove that the nurse owed a duty to the client, that the nurse breached the duty, and that as result harm was caused to person or property. 

  • Option A: Replacing fluid volume is not the issue; furthermore, the goal of therapy is to reduce total body fluid. Diuretics play a crucial role in treating edema and hypertension by causing the induction of a negative balance of solute and water. Loop diuretics are physiologically the most potent family of diuretics. 
  • Option B: The client has an increased risk of hypotension because hypotension is a common adverse effect of bumetanide, this is the second dose within 4 hours, and the client has heart failure. 
  • Option C: The client can prove that the nurse did not protect him by failing to provide adequate teaching and perform correct and timely nursing interventions after administering the bumetanide. 
  • Option D: After the first 15-minute check, the nurse should continue monitoring the client to ensure compliance with safety measures. Blood pressure, uric acid, jugular venous pressure, blood glucose, electrolytes, blood urea nitrogen/serum creatinine, and urine output must all need monitoring in patients taking bumetanide.
  • Option E: No data indicate that the dose of bumetanide, a loop diuretic, was excessive. To control edema, a staggering dosing schedule or a 3 to 4 times daily dosing schedule with half-day rest intervals in between is recommended to increase tolerability and efficacy. It is the safest and most effective method for the continued control of edema.
  • Option F: However, because this medication can cause hypotension, especially after a repeat dose, the nurse should instruct the client to remain in bed and provide him with a urinal. It may be difficult for the client to prove that the second dose of bumetanide caused the injury.

NURSESLABS-SATA-2-016

A nurse develops a plan of care for a client following a lumbar puncture. Which interventions should be included in the plan? Select all that apply.

  • A. Monitor the client’s ability to void.
  • B. Maintain the client in a flat position.
  • C. Restrict fluid intake for a period of 2 hours.
  • D. Monitor the client’s ability to move the extremities.
  • E. Inspect the puncture site for swelling, redness, and drainage.
  • F. Maintain the client on a nothing-by-mouth (NPO) status for 24 hours.

Correct Answer: A, B, D, & E.

Lumbar puncture, also known as a spinal tap, is an invasive procedure where a hollow needle is inserted into the space surrounding the subarachnoid space in the lower back to obtain samples of cerebrospinal fluid (CSF) for qualitative analysis.

  • Option A: The nurse should monitor the client’s ability to void. Take vital signs, measure intake and output, and assess neurologic status at least every 4 hours for 24 hours to allow further evaluation of the patient’s condition.
  • Option B: Following a lumbar puncture, the client remains flat in bed for 6 to 24 hours, depending on the health care provider’s prescriptions. He or she may turn from side to side as long as the head is not elevated.
  • Option C: A liberal fluid intake is encouraged to replace cerebrospinal fluid removed during the procedure unless contraindicated by the client’s condition. An increased amount of fluid intake (up to 3,000 ml in 24 hours) will replace CSF removed during the lumbar puncture.
  • Option D: The nurse should monitor the client’s ability to move the extremities. A feeling of tingling sensation and numbness in the lower back and legs is felt temporarily.
  • Option E: The nurse checks the puncture site for redness and drainage. Signs of CSF leakage include positional headaches, nausea and vomiting, neck stiffness, photophobia (sensitivity to light), sense of imbalance, tinnitus (ringing in the ear), and phonophobia (sensitivity to sound). 

Questions and rationale from Nurseslabs.com Feel free to print or share and link back to us! For more practice questions, please visit our Nursing Test Bank [https://nurseslabs.com/nursing-test-bank]

NURSESLABS-SATA-2-017

A nurse is assisting a gastroenterologist in caring for a client with complaints of epigastric pain. The nurse is explaining the role of the gastric glands in the fundus and body of the stomach which secrete intrinsic factor and hydrochloric acid. The nurse is correct when stating which of these substances as those needed in the GI tract. Select all that apply.

  • A. Vitamin B 12 absorption.
  • B. Emulsifying fats.
  • C. Dissolving food fibers.
  • D. Killing microorganisms.
  • E. Activating the enzyme pepsin.
  • F. Vitamin B 6 absorption.

Correct Answer: A, C, D, & E.

Hydrochloric acid (HCl), the main constituent of gastric acid, is secreted by parietal cells. The hydrogen (H) and chloride (Cl) components of HCl are secreted separately by hydrogen/potassium ATPase pumps and chloride channels in the stomach. Pepsinogen, a proenzyme for pepsin, is secreted by chief cells.

  • Option A: Intrinsic factor is needed for vitamin B12 absorption. Approximately 1.2% of vitamin B12 is absorbed passively without the help of intrinsic factors. If a patient receives the oral formulation at high doses, this passive absorption is sufficient to replenish vitamin B12 deficiency.
  • Option B: Bile is the substance secreted from the gallbladder to emulsify fats as they are consumed. Once the food is present in the duodenum (especially fatty food), the I cells are stimulated to secrete CCK which in turn causes gallbladder wall contraction as well as relaxation of the sphincter of Oddi. The bile then flows into the second part of the duodenum and causes emulsification of large fat droplets into small ones.
  • Option C: Hydrochloric acid is needed for dissolving food fibers. When pepsinogen and hydrochloric acid exist together in the gastric juice, pepsin takes its active form. Through the actions of pepsin and the squeezing properties of the stomach, the food bolus enters the intestines as a liquid mixture of partially digested food particles, called chyme.
  • Option D: Hydrochloric acid is needed for killing microorganisms. The acidic environment of the stomach is not only useful for protein denaturing but also for protection against potentially infectious agents. All material consumed by the body must pass through the stomach, making it an important defense against microbes. Many bacteria are killed or inhibited by the stomach’s acidity.
  • Option E: Hydrochloric acid is needed for activating the enzyme pepsin. Collectively, gastric acid creates an acidic environment that denatures proteins and activates the conversion of pepsinogen to pepsin. Pepsin breaks down proteins into smaller peptides, which may be further processed and later absorbed in the small intestine.
  • Option F: Vitamin B6, an essential nutrient, must be replaced daily because it is water-soluble and eliminated in urine. As a coenzyme, vitamin B6 is involved as a cofactor in over 100 enzymatic reactions including amino acid metabolism, carbohydrate metabolism, and lipid metabolism. It contributes to cognitive development via neurotransmitter synthesis, immune function via interleukin-2 production, and hemoglobin formation.

 NURSESLABS-SATA-2-018

A nurse is developing a care plan for a client with an injury to the frontal lobe of the brain. Which nursing interventions should be included as part of the care plan? Select all that apply.

  • A. Keep instructions simple and brief because the client will have difficulty concentrating.
  • B. Speak clearly and slowly because the client will have difficulty hearing.
  • C. Assist with bathing because the client will have vision disturbances.
  • D. Orient the client to person, place, and time as needed because of memory problems.
  • E. Assess vital signs frequently because vital bodily functions are affected.

Correct Answer: A & D.

Damage to the frontal lobe affects personality, memory, reasoning, concentration, and motor control of speech. The cortex of the frontal lobe is the largest of the four, and in many ways the lobe which participates most in making us human.

  • Option A: The prefrontal cortex is known to be the higher-order association center of the brain as it is responsible for decision making, reasoning, personality expression, maintaining social appropriateness, and other complex cognitive behaviors. 
  • Option B: Damage to the temporal lobe, not the frontal lobe, causes hearing and speech problems. Another study divides the temporal area into 4 major subregions: a) dorsal, mostly language and auditory/somatosensory networks b) ventromedial, mostly visual network c) medial, connected to paralimbic structures and d) anterolateral, associated with a default-semantic network. These areas have many important functions such as processing of language, social cues, and emotions, facial recognition (auditory and visual aspects), emotional processing of different stimuli (auditory, olfactory and visual) and theory of mind.
  • Option C: Damage to the occipital lobe causes vision disturbances. The occipital lobe is the visual processing area of the brain. It is associated with visuospatial processing, distance and depth perception, color determination, object and face recognition, and memory formation.
  • Option D: Research has proven that the dominant (left) superior frontal gyrus is a key component in the neural network of working memory as well as spatial processing.Research has proven that the dominant (left) superior frontal gyrus is a key component in the neural network of working memory as well as spatial processing.
  • Option E: Damage to the brain stem affects vital functions. The brainstem is the structure that connects the cerebrum of the brain to the spinal cord and cerebellum. It is composed of four sections in descending order: the diencephalon, midbrain, pons, and medulla oblongata. It is responsible for many vital functions of life, such as breathing, consciousness, blood pressure, heart rate, and sleep.

NURSESLABS-SATA-2-019

A nurse has reinforced instructions to the client with hyperparathyroidism regarding home care measures related to exercise. Which statement by the client indicates a need for further instruction? Select all that apply.

  • A. “I enjoy exercising but I need to be careful.”
  • B. “I need to pace my activities throughout the day.”
  • C. “I need to limit playing football to only the weekends.”
  • D. “I should gauge my activity level by my energy level.”
  • E. “I should exercise in the evening to encourage a good sleep pattern.”

Correct Answer: C & E.

Primary hyperparathyroidism (PHPT) is a disorder of one or more of the parathyroid glands . The parathyroid gland(s) becomes overactive and secretes excess amounts of parathyroid hormone (PTH). As a result, the blood calcium rises to a level that is higher than normal (called hypercalcemia). An elevated calcium level can cause many short-term and long-term complications.

  • Option A: The client should plan for at least 30 minutes of walking each day to support calcium movement into the bones. Every person is different in terms of their fitness level and the severity of their disease. That’s why it is important to start slowly and gradually increase the intensity of the exercise routine over time.
  • Option B: The client with hyperparathyroidism should pace activities throughout the day and plan for periods of uninterrupted rest.
  • Option C: The client should be instructed to avoid high-impact activity or contact sports such as football. Research has shown that PTH levels tend to increase following either high-intensity exercise over a long period (greater than 50 minutes) or low-intensity exercise over a very long period (around five hours).
  • Option D: The client should be instructed to use energy levels as a guide to activity. Data suggested that short-duration exercise at high levels of exertion, or low-intensity exercise over a moderate time period (50 minutes), did not appear to affect PTH levels.
  • Option E: Exercising late in the evening may interfere with restful sleep. However, it can aid with sleeping and increasing the energy levels, as long as it isn’t overdone and if done at the right time.

NURSESLABS-SATA-2-020

A nurse in a medical unit is caring for a client with heart failure. The client suddenly develops extreme dyspnea, tachycardia, and lung crackles, and the nurse suspects pulmonary edema. The nurse immediately notifies the registered nurse and expects which interventions to be prescribed? Select all that apply.

  • A. Administering oxygen
  • B. Inserting a Foley catheter
  • C. Administering furosemide (Lasix)
  • D. Administering morphine sulfate intravenously
  • E. Transporting the client to the coronary care unit
  • F. Placing the client in a low Fowler’s side-lying position

Correct Answer: A, B, C, & D.

A pulmonary edema is a life-threatening event that can result from severe heart failure. In pulmonary edema, the left ventricle fails to eject sufficient blood, and pressure increases in the lungs because of the accumulated blood. 

  • Option A: Oxygen is always prescribed. Supplemental oxygen increases oxygen availability to the myocardium and can help relieve symptoms of hypoxemia, ischemia, and subsequent activity intolerance (Giordano, 2005; Haque et al., 1996). The need is based on the degree of pulmonary congestion and resulting hypoxia.
  • Option B: A Foley catheter is inserted to accurately measure output. Urine output may be scanty and concentrated (especially during the day) because of reduced renal perfusion. Recumbency favors diuresis; therefore, urine output may be increased at night and/or during bed rest.
  • Option C: Furosemide, a rapid-acting diuretic, will eliminate accumulated fluid. Evaluate urine output in response to diuretic therapy. The focus is on monitoring the response to the diuretics rather than the actual amount voided.
  • Option D: Intravenously administered morphine sulfate reduces venous return (preload), decreases anxiety, and reduces the work of breathing. The use of morphine should be reserved for patients with myocardial ischemia who are refractory to drugs that favorably alter myocardial oxygen supply and demand.
  • Option E: Transporting the client to the coronary care unit is not a priority intervention. In fact, this may not be necessary at all if the client’s response to treatment is successful.
  • Option F: The client is placed in a high Fowler’s position to ease the work of breathing. Allows for better chest expansion, thereby improving pulmonary capacity. In this position, the venous return to the heart is reduced, pulmonary congestion is alleviated, and pressure on the diaphragm is minimized.

Select All That Apply NCLEX Practice #2

NURSESLABS-SATA-2-001

A nurse is admitting a client with a possible diagnosis of chronic bronchitis. The nurse collects data from the client and notes which of the following signs supports this diagnosis? Select all that apply.

  • A. Scant mucus
  • B. Early onset cough
  • C. Marked weight loss
  • D. Purulent mucus production
  • E. Mild episodes of dyspnea

Correct Answer: B, D, & E.

Key features of pulmonary emphysema include dyspnea that is often marked, late cough (after the onset of dyspnea), scant mucus production, and marked weight loss. By contrast, chronic bronchitis is characterized by an early onset of cough (before dyspnea), copious purulent mucus production, minimal weight loss, and milder severity of dyspnea.

  • Option A: Most patients with emphysema present with very nonspecific symptoms of chronic shortness of breath and cough with or without sputum production. As the disease process advances, the shortness of breath and cough progressively get worse.
  • Option B: The most common symptom of patients with chronic bronchitis is a cough. The history of a cough typical of chronic bronchitis is characterized to be present for most days in a month lasting for 3 months with at least 2 such episodes occurring for 2 years in a row. The characteristic cough of bronchitis is caused by the copious secretion of mucus in chronic bronchitis.
  • Option C: As COPD advances, patients can lose significant body weight due to systemic inflammation and increased energy spent in the work of breathing. Also, there are frequent intermittent exacerbations as the obstruction of the airways increases.
  • Option D: The airways become clogged by debris and this further increases the irritation. A productive cough with sputum is present in about 50% of patients. The sputum color may vary from clear, yellow, green, or at times blood-tinged. The color of the sputum may be dependent on the presence of secondary bacterial infection.
  • Option E: During an acute exacerbation of chronic bronchitis, the bronchial mucous membrane becomes hyperemic and edematous with diminished bronchial mucociliary function. This, in turn, leads to airflow impediment because of luminal obstruction to small airways.

NURSESLABS-SATA-2-002

A nurse, assigned to a client with emphysema, is providing a shift report. Which nursing interventions would be appropriate to include? Select all that apply.

  • A. The nurse should reduce fluid intake to less than 850 ml per shift.
  • B. The nurse should teach diaphragmatic, pursed-lip breathing.
  • C. The nurse should administer low-flow oxygen.
  • D. The nurse should keep the client in a supine position as much as possible. 
  • E. The nurse should encourage alternating activity with rest periods.
  • F. The nurse should teach the use of postural drainage and chest physiotherapy.

Correct Answer: B, C, E, & F.

Emphysema is the most severe form of COPD, characterized by recurrent inflammation that damages and eventually destroys alveolar walls to create large blebs or bullae (air spaces) and collapsed bronchioles on expiration (air-trapping).

  • Option A: Fluid intake should be increased to 3,000 ml/day, if not contraindicated, to liquefy secretions and facilitate their removal. Provide warm or tepid liquids. Recommend the intake of fluids between, instead of during, meals. Using warm liquids may decrease bronchospasm. Fluids during meals can increase gastric distension and pressure on the diaphragm.
  • Option B: Diaphragmatic, pursed-lip breathing strengthens respiratory muscles and enhances oxygenation in clients with emphysema. This provides the client with some means to cope with or control dyspnea and reduce air-trapping.
  • Option C: Low-flow oxygen should be administered because a client with emphysema has chronic hypercapnia and a hypoxic respiratory drive. Administering humidified oxygen prevents drying out the airways, decreases convective moisture losses, and improves compliance.
  • Option D: The client should be placed in high Fowler’s position to improve ventilation. Elevation of the head of the bed facilitates respiratory function by use of gravity; however, the client in severe distress will seek the position that most eases breathing.
  • Option E: Alternating activity with rest allows to perform activities without excessive distress. During severe, acute or refractory respiratory distress, the patient may be totally unable to perform basic self-care activities because of hypoxemia and dyspnea. Rest interspersed with care activities remains an important part of the treatment regimen.
  • Option F: If the client has difficulty mobilizing copious secretions, the nurse should teach the client and family members how to perform postural drainage and chest physiotherapy. These techniques will prevent possible aspirations and prevent any untoward complications.

NURSESLABS-SATA-2-003

A nurse is assigned to care for a client with a peripheral IV infusion. The nurse is providing hygiene care to the client and would avoid which of the following while changing the client’s hospital gown? 

A. Using a hospital gown with snaps at the sleeves

B. Disconnecting the IV tubing from the catheter in the vein

C. Checking the IV flow rate immediately after changing the hospital gown

D. Putting the bag and tubing through the sleeve, followed by the client’s arm

E. Keeping splint soiled by blood or fluid leakage

Correct Answer: B & E.

Changing a patient’s hospital gown is needed to maintain their cleanliness and the feeling of freshness.

  • Option A: A kimono-inspired gown opens in the front and uses a system of ties and snaps at essential access points for easy treatment and monitoring.Top snaps offer upper back access. Wide sleeves and side snaps provide easy access for an I.V., and are MRI-compatible.
  • Option B: The tubing should not be removed from the IV catheter. With each break in the system, there is an increased chance of introducing bacteria into the system, which can lead to infection. 
  • Option C: The flow rate should be checked immediately after changing the hospital gown, because the position of the roller clamp may have been affected during the change. Count the rate of flow of the infusion to make sure it is correct before leaving the bedside.
  • Option D: Holding the container above the client’s arm, slide the sleeve up over the container to remove the used gown. Place the clean gown sleeve for the arm with the infusion over the container as if it were an extension of the client’s arm, from the inside of the gown to the sleeve cuff.
  • Option E: IV board/splints are recommended to secure PIVC placed in or adjacent to areas of flexion. This will adequately immobilize the joint and minimize the risk of venous damage resulting from flexion. Splints should be Inspected at least daily and change if soiled by blood or fluid leakage. 

NURSESLABS-SATA-2-004

A nurse is caring for a client who underwent surgical repair of a detached retina in the right eye. Which nursing interventions should the nurse perform? Select all that apply.

  • A. Place the client in a prone position.
  • B. Approach the client from the left side.
  • C. Encourage deep breathing and coughing.
  • D. Discourage bending down.
  • E. Orient the client to the environment.
  • F. Administer a stool softener.

Correct Answer: B, D, E, & F.

Retinal detachment is a serious condition of the eye in which the retina stops receiving oxygen. Retinal detachment repair is a surgery that is used to restore circulation to the retina and preserve vision.

  • Option A: The client should lie on the back or on the unaffected side to reduce intraocular pressure in the affected eye. The bubble will move to the front of the eye and press against the lens instead of the retina.
  • Option B: The nurse should approach the client from the left side—the unaffected side—to avoid startling. The client may have to wear a patch or shield over the eye for a day or more.
  • Option C: Allow the eye to heal. Don’t do things where there is sudden movement of the head. This includes moving quickly, lifting anything heavy, or doing activities such as cleaning or gardening.
  • Option D: The nurse should also discourage the client from bending down, deep breathing, hard coughing and sneezing, and other activities that can increase intraocular pressure during the postoperative period. 
  • Option E: The client should be oriented to the environment to reduce the risk of injury. More than 90 percent of detachments can be repaired. In less than 10 percent of detachments that cannot be repaired, the patient will have either poor vision or no vision in that eye.
  • Option F: Stool softeners should be administered to discourage straining during defecation. Stool softeners help prevent constipation. Constipation causes the client to strain. This can increase pressure in your eye and cause damage.

NURSESLABS-SATA-2-005

A nurse is planning care for a client with hyperthyroidism. Which nursing interventions are appropriate? Select all that apply.

  • A. Instill isotonic eye drops as necessary.
  • B. Provide several small, well-balanced meals.
  • C. Provide rest periods.
  • D. Keep the environment warm.
  • E. Encourage frequent visitors and conversation.
  • F. Weigh the client daily.

Correct Answer: A, B, C, & F.

Hyperthyroidism may manifest as weight loss despite an increased appetite, palpitation, nervousness, tremors, dyspnea, fatigability, diarrhea or increased GI motility, muscle weakness, heat intolerance, and diaphoresis. A patient with hyperthyroidism classically presents with signs and symptoms that reflect this state of increased metabolic activity.

  • Option A: If the client has exophthalmos (a sign of hyperthyroidism), the conjunctivae should be moistened often with isotonic eye drops. This protects exposed cornea if the patient is unable to close eyelids completely because of edema or fibrosis of fat pads and/or exophthalmos.
  • Option B: Hyperthyroidism results in increased appetite, which can be satisfied by frequent, small, well-balanced meals. Encourage the client to eat and increase the number of meals and snacks. Give or suggest high-calorie foods that are easily digested.
  • Option C: The nurse should provide the client with rest periods to reduce metabolic demands. The client should be weighed daily to check for weight loss, a possible consequence of hyperthyroidism. 
  • Option D: Because metabolism is increased in hyperthyroidism, heat intolerance may result. Therefore, the nurse should provide a cool environment, not a warm one, to promote client comfort.
  • Option E: Because metabolism is increased in hyperthyroidism, excitability may result.  The nurse should provide a quiet environment, not a busy one. Provide for a quiet environment; cool room, decreased sensory stimuli, soothing colors, quiet music.

NURSESLABS-SATA-2-006

A nurse is assisting with planning care for a client with an internal radiation implant. Which of the following should be included in the plan of care? Select all that apply.

  • A. Wearing gloves when emptying the client’s bedpan
  • B. Keeping all linens in the room until the implant is removed
  • C. Wearing a film (dosimeter) badge when in the client’s room
  • D. Wearing a lead apron when providing direct care to the client
  • E. Placing the client in a semi private room at the end of the hallway

Correct Answer: A, B, C, & D.

A private room with a private bath is essential if a client has an internal radiation implant. This is necessary to prevent the accidental exposure of other clients to radiation. The remaining options identify interventions that are necessary for a client with a radiation device.

  • Option A: Use appropriate contact precautions when handling the client’s linens, urine, saliva, perspiration, vomit or feces. They may be potentially radioactive. Wear gloves and gowns to protect yourself.
  • Option B: Inform the client that he is not radioactive instead that the inserted implants are. This is why certain precautions have to be taken, to reduce exposure to others.
  • Option C: Wear a detection badge to determine how much exposure you have had to your client’s radioactive implants in order to prevent overexposure.
  • Option D: Wear a lead apron when caring for the client. Always use the principle of time, distance, and shielding. spend as little time in the room as you can, stay far away and protect yourself with appropriate clothing.
  • Option E: Place the client in a private room with his bed in the center of the room. The radiation may be able to penetrate the walls if they are not lead lined, therefore having the bed in the center of the room reduces the risk of radiation leak outside of the room.

NURSESLABS-SATA-2-007

The nurse is caring for a client after a supratentorial craniotomy in which a large tumor was removed from the left side. Choose the positions in which the nurse can safely place the client. Select all that apply.

  • A. On the left side
  • B. With the neck flexed
  • C. Supine on the left side
  • D. With extreme hip flexion
  • E. In a semi-Fowler’s position
  • F. With the head in a midline position

Correct Answer: E and F.

The postoperative course of clients undergoing surgery for cranial tumors depends on the extent and location of surgery, intraoperative issues and complications, and postoperative complications.

  • Option A: If a large tumor has been removed, the client should be placed on the non operative side to prevent the displacement of the cranial contents.
  • Option B: Avoid extreme flexion of upper legs or flexion of the neck. The nurse may place the patient on his side to promote airway and facilitate drainage of secretions.
  • Option C: The HOB after supratentorial craniotomy should be at least at 30 degrees. Avoidance of prolonged pressure directly on the incision will prevent breakdown or added discomfort.
  • Option D: Depending on the extent of surgery and immediate postoperative condition, the client may start mobilizing the following day, depending on age, but sitting up, sitting in a chair, standing, and eventually walking if able.
  • Option E: Clients who have undergone supratentorial surgery should have the head of the bed elevated 30 degrees to promote venous drainage from the head.
  • Option F: The client is positioned to avoid extreme hip or neck flexion, and the head is maintained in a midline, neutral position.

NURSESLABS-SATA-2-008

A client has a tumor of the posterior pituitary gland. A nurse planning care should include which nursing interventions? Select all that apply.

  • A. Weigh the client daily.
  • B. Restrict fluids.
  • C. Measure urine specific gravity.
  • D. Encourage intake of coffee or tea.
  • E. Monitor intake and output.

Correct Answer: A, C, & E.

The pituitary gland is divided into the anterior and posterior sections with each section secreting specific hormones. Tumors of the posterior pituitary gland can lead to diabetes insipidus due to deficiency of vasopressin, also called antidiuretic hormone (ADH). Decreased ADH reduces the kidneys’ ability to concentrate urine, resulting in excessive urination, thirst, and fluid intake. 

  • Option A: To monitor fluid balance, the nurse should weigh the client daily, measure urine specific gravity, and monitor intake and output. Weight loss occurs with excessive fluid loss.
  • Option B: The nurse should also encourage fluids to keep intake equal to output and prevent dehydration. Provide easily accessible fluid sources, keeping adequate fluids at bedside. This encourages fluid intake.
  • Option C: Monitor urine specific gravity. This may be 1.005 or less. Monitor serum and urine osmolality. Urine osmolality will be decreased and serum osmolality will increase. Monitor urine and serum sodium levels. The patient with DI has decreased urine sodium levels and hypernatremia.
  • Option D: Coffee, tea, and other fluids that have a diuretic effect should be avoided. Allow the patient to drink water at will. Patients with intact thirst mechanisms may maintain fluid balance by drinking huge quantities of water to compensate for the amount they urinate. Patients prefer cold or ice water.
  • Option E: Monitor intake and output. Report urine volume greater than 200 mL for each of 2 consecutive hours or 500 mL in a 2-hour period. With DI, the patient voids large urine volumes independent of the fluid intake. Urine output ranges from 2 to 3 L/day with renal DI to greater than 10 L/day with central DI.

NURSESLABS-SATA-2-009

A nurse is caring for a client with a healthcare-associated infection caused by methicillin-resistant Staphylococcus aureus who is on contact precautions. The nurse prepares to provide colostomy care to the client. Which of the following protective items will be required to perform this procedure? Select all that apply.

  • A. Gloves
  • B. Goggles
  • C. A gown
  • D. Shoe protectors

Correct Answer: A, B, & C.

MRSA infection is one of the leading causes of hospital-acquired infections and is commonly associated with significant morbidity, mortality, length of stay, and cost burden. Prevention and control of MRSA infections include necessary infection-control steps like strict hand hygiene and adequate contact precautions.

  • Option A: Contact precautions require the use of gloves. Hand hygiene means washing hands with soap and water or an alcohol-based cleanser before and after contact with patients who have MRSA infections.
  • Option B: Goggles are worn to protect the mucous membranes of the eye during interventions that may produce splashes of blood, body fluids, secretions, and excretions. Besides the standard precautions, the CDC recommends contact precautions. The patient should be in an isolated room if available.
  • Option C: A gown should be worn if direct client contact is anticipated. Contact precautions include the use of gowns, gloves, and possibly masks during clinical encounters with patients with MRSA infection.
  • Option D: Shoe protectors are not necessary. Instead, everyone should gown and glove when coming into contact with the patient.  The transport of MRSA patients should be minimized and dedicated medicated equipment should be used on them.

NURSESLABS-SATA-2-010

A nurse is caring for a client with a hiatal hernia. The client complains of abdominal and sternal pain after eating. The pain makes it difficult for the client to sleep. Which instructions should the nurse recommend when teaching this client? Select all that apply.

  • A. Avoid constrictive clothing.
  • B. Lie down for 30 minutes after eating.
  • C. Decrease the intake of caffeine and spicy foods.
  • D. Eat three meals per day.
  • E. Sleep with the upper body elevated.
  • F. Maintain a normal body weight.

Correct Answer: A, C, E, & F.

A hiatal hernia occurs when a portion of the stomach pushes through the diaphragm. A hiatal hernia may cause abdominal and sternal pain after eating. The discomfort is associated with reflux of gastric contents. 

  • Option A: To reduce gastric reflux, the nurse should instruct the client to avoid constrictive clothing. Instruct the client to avoid bending over, coughing, straining at defecations, and other activities that increase reflux.
  • Option B: To reduce gastric reflux, the nurse should instruct the client to remain upright for 2 hours after eating. Instruct to remain in upright position at least 2 hours after meals; avoiding eating 3 hours before bedtime. This helps control reflux and causes less irritation from reflux action into the esophagus.
  • Option C: To reduce gastric reflux, the nurse should instruct the client to avoid caffeine and spicy foods. Avoid foods and liquids, such as coffee and alcohol, that stimulate the secretion of stomach acids. Instruct the client to avoid highly seasoned food, acidic juices, alcoholic drinks, bedtime snacks, and foods high in fat.
  • Option D: To reduce gastric reflux, the nurse should instruct the client to eat small, frequent meals. Encourage small frequent meals of high calories and high protein foods. Small and frequent meals are easier to digest.
  • Option E: To reduce gastric reflux, the nurse should instruct the client to sleep with the upper body elevated. Elevating the head of the bed six inches to prevent the reflux of stomach contents into the esophagus.
  • Option F: To reduce gastric reflux, the nurse should instruct the client to lose weight, if obese. Accurately measure the client’s weight and height for baseline data. Identify the amount of weight loss needed for optimal body size and frame. This provides a basis for dietary planning.

NURSESLABS-SATA-2-011

A nurse is caring for a client with diabetic ketoacidosis and documents that the client is experiencing Kussmaul’s respirations. Based on this documentation, which of the following did the nurse most likely observe? Select all that apply.

  • A. Respirations that cease for several seconds
  • B. Respirations that are regular but abnormally slow
  • C. Respirations that are labored and increased in-depth and rate
  • D. Respirations that are abnormally deep, regular, and increased in rate
  • E. Respirations are rapid and shallow but as acidosis worsens, breathing gradually becomes deep, labored, and gasping

Correct Answer: D & E.

Kussmaul’s respiration is a deep and labored breathing pattern often associated with severe metabolic acidosis, particularly diabetic ketoacidosis (DKA) but also kidney failure. It is a form of hyperventilation. It results from stimulation of the respiratory center in the brain stem by low serum pH.

  • Option A: In apnea, respirations cease for several seconds. Apnea is the absence of breathing. This signals a life-threatening situation in which the patient will quickly succumb unless rescue breathing is instituted immediately. 
  • Option B: In bradypnea, respirations are regular but abnormally slow. Bradypnea is a respiratory rate that is lower than normal for age.
  • Option C: In hyperpnea, respirations are labored and increased in depth and rate. Hyperpnea in increased volume with or without an increased rate of breathing. Blood gasses are normal.
  • Option D: They are abnormally deep, regular, and increased in rate. As classically described, Kussmaul respirations are a deep, sighing respiratory pattern. Dr. Kussmaul actually described it as “air hunger.” Kussmaul’s respiratory pattern occurs due to increased tidal volume with or without an increased respiratory rate.
  • Option E: In metabolic acidosis, breathing is first rapid and shallow but as acidosis worsens, breathing gradually becomes deep, labored, and gasping. This is probably the most important of the abnormal respiratory patterns.

NURSESLABS-SATA-2-012

Which nursing interventions are appropriate for a client recovering from surgery for retinal detachment? Select all that apply.

  • A. Monitor for hemorrhage.
  • B. Administer eye medications.
  • C. Maintain the eye patch or shield.
  • D. Assist with activities of daily living.
  • E. Encourage coughing and deep breathing.
  • F. Educate regarding symptoms of retinal detachment.

Correct Answer: A, B, C, D, & F.

Retinal detachments constitute a serious ocular condition and can lead to permanent vision loss. When the retina, the neurosensory layer, detaches from the back of the eye, it loses its oxygen and nutrient supply leading to the death of the tissue.

Questions and rationale from Nurseslabs.com Feel free to print or share and link back to us! For more practice questions, please visit our Nursing Test Bank [https://nurseslabs.com/nursing-test-bank]

  • Option A: Hemorrhage is also a risk post surgery. Spontaneous choroidal hemorrhage often occurs either intraoperatively or in the immediate postoperative period.
  • Option B: Eye medications are prescribed postoperatively. Zylet drops minimizes inflammation and the risk of infection following retina surgery. Alphagan drop is used to control eye pressure. Atropine minimizes pain caused by swelling and inflammation, following retina surgery. 
  • Option C: An eye patch or shield is applied to protect the eye and prevent any further detachment. Protective shields should be worn for at least a week and guidelines should be followed closely to ensure incisions heal properly without disruption.
  • Option D: Positioning, activity restrictions, and eye patches hinder the client in the performance of activities of daily living, and the client needs the nurse’s assistance with these activities. Activity must be restricted, and patients should refrain bending and heavy lifting (2-10 lb. limit).
  • Option E: Coughing is not encouraged because this can increase intraocular pressure and harm the client. Coughing has been implicated in the development of spontaneous choroidal hemorrhage in the immediate postoperative period. The mechanism is felt to be an increase in episcleral venous pressure, resulting in a rupture of the wall of the ciliary vessels.
  • Option F: Educating the client regarding symptoms is necessary because the client is at risk for subsequent retinal detachment. Patients should be educated not to wait to be seen by an ophthalmologist or optometrist if they are having changes in their vision. Vision loss from a retinal detachment can be permanent, and patients should be made aware of this.

NURSESLABS-SATA-2-013

A nurse is caring for a client with leukemia and notes that the client has flat neck and hand veins. The nurse suspects hyponatremia. What additional signs would the nurse expect to note in this client if hyponatremia is present?

  • A. Intense thirst
  • B. Slow bounding pulse
  • C. Dry mucous membranes
  • D. Poor skin turgor
  • E. Postural blood pressure changes

Correct Answer: D & E.

Hyponatremia is defined as a serum sodium concentration of less than 135 mEq/L but can vary to a small extent in different laboratories. Hyponatremia is a common electrolyte abnormality caused by an excess of total body water when compared to total body sodium content.

  • Option A: Intense thirst is seen in clients with hypernatremia. The imbalance of water intake and excretion causes hyponatremia or hypernatremia. Water intake is regulated by the thirst mechanism where osmoreceptors in the hypothalamus trigger thirst when body osmolality reaches 295 mOsm/kg.
  • Option B: A slow, bounding pulse is not indicative of hyponatremia. In a client with hyponatremia, a rapid thready pulse is noted. Reduced dietary sodium intake (sodium reduction) increases heart rate in some studies of animals and humans. As heart rate is independently associated with the development of heart failure and increased risk of premature death a potential increase in heart rate could be a harmful side-effect of sodium reduction.
  • Option C: Dry mucous membranes are seen in clients with hypernatremia. There are other signs that could lead to a diagnosis of hyponatremia. These are generally dehydration-related symptoms, including dry mucous membranes and reduced elasticity of the skin.
  • Option D: If the skin of the arm, calf, or thigh is pinched in healthy subjects, it will immediately return to its normally flat state when the pinch is released. This elastic property, called turgor, is lost when there is a loss in the interstitial fluid.
  • Option E: Postural blood pressure changes occur in the client with hyponatremia. Antidiuretic hormone (ADH) is released when there is a reduction of 15% or more of the EABV. This occurs with hypovolemia (e.g., vomiting, diarrhea), decreased cardiac output (e.g., heart failure), or vasodilation (e.g., cirrhosis). 

NURSESLABS-SATA-2-014

A nurse is caring for a group of clients who are taking herbal medications at home. Which of the following clients should be instructed not to take herbal medications? Select all that apply.

  • A. A 60-year-old male client with rhinitis
  • B. A 24-year-old male client with a lower back injury
  • C. An 8-year-old uncircumcised male client with a urinary tract infection
  • D. A 10-year-old female client with a urinary tract infection
  • E. A 45-year-old female client with a history of migraine headaches

Correct Answer: C & D.

Herbal supplements are products derived from plants and/or their oils, roots, seeds, berries or flowers. Herbal supplements have been used for many centuries. They are believed to have healing properties.

  • Options A and E: Feverfew (Tanacetum parthenium) is believed to prevent and treat migraine, arthritis and allergies. Feverfew can interfere with blood clotting when taken internally.
  • Option B: Arnica (Arnica montana) is applied externally to reduce pain from bruising, aches, and sprains, and to relieve constipation. Arnica is potentially toxic to the heart and can raise blood pressure if taken internally.
  • Options C and D: Children should not be given herbal therapies, especially in the home and without professional supervision.

NURSESLABS-SATA-2-015

A nurse is caring for an infant with a diagnosis of tetralogy of Fallot. The infant suddenly becomes cyanotic and the oxygen saturation reading drops to 60%. Choose the interventions that the nurse should perform. Select all that apply.

  • A. Call a code blue.
  • B. Notify the registered nurse.
  • C. Place the infant in a prone position.
  • D. Prepare to administer morphine sulfate.
  • E. Prepare to administer intravenous fluids.
  • F. Prepare to administer 100% oxygen by face mask.

Correct Answer: B, D, E, & F.

The child who is cyanotic with oxygen saturations dropping to 60% is having a hypercyanotic episode. Hypercyanotic episodes often occur among infants with tetralogy of Fallot, and they may occur among infants whose heart defect includes the obstruction of pulmonary blood flow and communication between the ventricles. 

  • Option A: There is no reason to call a code blue unless respirations cease. The outcomes for TOF patients are good but after two decades a significant number of them will require pulmonary valve replacement. Unlike in the past, today there is a percutaneous method of implanting a pulmonary valve, but long term results are not known.
  • Option B: The registered nurse is notified, who will then contact the health care provider. The diagnosis and management of TOF is with an interprofessional team that includes a pediatrician, pediatric cardiologist, cardiac surgeon, and radiologist.
  • Option C: If a hypercyanotic episode occurs, the infant is placed in a knee-chest position immediately. The knee-chest position improves systemic arterial oxygen saturation by decreasing venous return so that smaller amounts of highly saturated blood reach the heart. Toddlers and children squat to get into this position and relieve chronic hypoxia. 
  • Option D: Additional interventions include administering morphine sulfate as prescribed. The most common subsequent treatment is to administer morphine, which calms the patient, resolves the hyperpnea, normalizes the systemic venous return, and increases the partial pressure of oxygen in the blood.
  • Option E: Tet spells require a rapid and aggressive approach including intravenous fluid bolus to improve the right ventricle filling and pulmonary flow, intravenous beta blockers to help improve the right ventricle outflow obstruction by relaxing the muscle, and intravenous phenylephrine to increase systemic afterload.
  • Option F: Administer oxygen therapy as prescribed. The failing heart may not be able to respond to increased oxygen demands. Oxygen saturation needs to be greater than 90%.

NURSESLABS-SATA-2-016

A nurse is collecting data on a client with severe preeclampsia. Choose the findings that would be noted in severe preeclampsia. Select all that apply.

  • A. Oliguria
  • B. Seizures
  • C. Contractions
  • D. Proteinuria 3+
  • E. Muscle cramps
  • F. Blood pressure 168/116 mm Hg

Correct Answer: A, D, & F.

Severe preeclampsia is characterized by blood pressure higher than 160/110 mmHg, proteinuria 3+ or higher, and oliguria. Preeclampsia is a hypertensive disease that occurs during pregnancy. This disease encompasses 2% to 8% of pregnancy-related complications, greater than 50,000 maternal deaths, and over 500,000 fetal deaths worldwide.

  • Option A: Women with preeclampsia commonly have transient oliguria (less than 100 mL over 4 hours) in labor or the first 24 hours postpartum. Patients at the severe end of the disease spectrum may have urine output <500 mL/24 hours.
  • Option B: Seizures (convulsions) are present in eclampsia and are not a characteristic of severe preeclampsia. The hallmark physical exam finding for eclampsia is generalized tonic-clonic seizures, which typically last 60 to 90 seconds in duration. A postictal state is often present after seizure activity.
  • Option C: A pregnant woman should immediately call her health care provider if any of the signs or symptoms of severe disease develop, or if she has decreased fetal activity, vaginal bleeding, abdominal pain, or frequent uterine contractions. The only cure for preeclampsia is delivery of the fetus and placenta.
  • Option D: Proteinuria in preeclampsia can be defined as any of the following [2]: ≥0.3 g protein in a 24-hour urine specimen. The completeness of the 24-hour urine collection can be estimated from creatinine excretion, which should be 15 to 20 mg/kg (133 to 177 micromol/kg) of lean body weight in women.
  • Option E: Muscle cramps are not findings noted in severe preeclampsia, although the client is monitored for these occurrences. Leg cramps may be caused by the additional weight gain of pregnancy and changes in circulation. Pressure from the growing baby may also be placed on the nerves and blood vessels that go to the legs. This pressure or pinching may be the cause of the leg cramps.
  • Option F: Patients with a systolic blood pressure of 140 mmHg or greater, or a diastolic pressure of 90 mmHg or greater, should increase suspicion for preeclampsia. In patients at greater than 20 weeks gestation, blood pressure readings on two measurements at least 4 hours apart should be evaluated with further diagnostic workup.

NURSESLABS-SATA-2-017

A nurse is monitoring a client with Graves’ disease for signs of thyrotoxicosis (thyroid storm). Which of the following signs and symptoms, if noted in the client, will alert the nurse to the presence of this crisis? Select all that apply.

  • A. Bradycardia
  • B. Fever
  • C. Sweating
  • D. Agitation
  • E. Pallor

Correct Answer: B, C, & D.

Thyrotoxic crisis (thyroid storm) is an acute, potentially life-threatening state of extreme thyroid activity that represents a breakdown in the body’s tolerance to a chronic excess of thyroid hormones. The clinical manifestations include fever greater than 100° F, severe tachycardia, flushing and sweating, and marked agitation and restlessness. Delirium and coma can occur.

  • Option A: Sinus tachycardia is the most common cardiac rhythm problem, but atrial fibrillation may occur and is frequently seen with advanced patient age, valvular disease, and coronary artery disease. Thyroid hormone causes increased expression of myocardial sarcoplasmic reticulum calcium-dependent ATP, increasing heart rate and myocardial contractility with the net effect of increased cardiac output.
  • Option B: In rare cases, patients present in thyroid storm with tachycardia, fever, altered mental status, agitation, features of cardiac failure, and impaired liver function. In subacute thyroiditis, patients have a history of recent upper respiratory illness and generally present with fever, neck pain and swelling with a firm and tender thyroid gland. 
  • Option C: Patients with thyrotoxicosis most commonly present with signs and symptoms related to excess thyroid hormone including weight loss with a normal or increased appetite, heat intolerance with increased sweating, palpitations, tremor, anxiety, proximal muscle weakness, alopecia and increased fatigability.
  • Option D: On physical exams, patients are often cachectic, hyperthermic, diaphoretic, and anxious. It is common for older patients to manifest fewer of the typical clinical manifestations and instead present with depression, fatigue, and weight loss, also known as apathetic thyrotoxicosis.
  • Option E: Hyperthyroidism increases blood flow in the extremities, which often causes the face to flush and the palms to turn red. Hypothyroidism produces the opposite effect and can leave the face pale.

NURSESLABS-SATA-2-018

A nurse is monitoring a group of clients for acid-base imbalances. Which clients are at highest risk for metabolic acidosis? Select all that apply.

  • A. Severely anxious client
  • B. Pneumonia client
  • C. Diabetic mellitus client
  • D. Malnourished client
  • E. Asthma client
  • F. Renal failure client

Correct Answer: C, D, & F.

Diabetes mellitus, malnutrition, and renal failure lead to metabolic acidosis because of the increasing acids in the body. Options A, B, and E are respiratory problems, not metabolic, and result in either respiratory acidosis or respiratory alkalosis.

  • Option A: Alveolar hyperventilation leads to a decreased partial pressure of arterial carbon dioxide (PaCO2). In turn, the decrease in PaCO2 increases the ratio of bicarbonate concentration to PaCO2 and, thereby, increases the pH level; thus the descriptive term respiratory alkalosis.
  • Option B: Any condition which decreases pulmonary compliance causes a sensation of dyspnea. Respiratory alkalosis is commonly found in patients with asthma, pneumonia & pulmonary embolism.
  • Option C: Diabetic ketoacidosis (DKA) is a widely known acute metabolic complication of diabetes mellitus (DM), which can be potentially fatal. It is not difficult to diagnose when a patient with DM comes with symptoms such as coma, fruity breath, hyperglycemia, acidosis, and tachypnea.
  • Option D: Metabolic acidosis, a common condition in patients with renal failure, may be linked to protein-energy malnutrition (PEM) and inflammation, together also known as malnutrition-inflammation complex syndrome (MICS).
  • Option E: Acute asthmatic crisis is usually accompanied by hyperventilation and hypocapnia with respiratory alkalosis. However, it seems that mild, asymptomatic asthma is also associated with hypocapnia.
  • Option F: The buildup of acid in the body due to kidney disease or kidney failure is called metabolic acidosis. When the body fluids contain too much acid, it means that the body is either not getting rid of enough acid, is making too much acid, or cannot balance the acid in your body.

 NURSESLABS-SATA-2-019

The nurse is preparing a teaching plan for a client who is undergoing cataract extraction with an intraocular implant. Which home care measures will the nurse include in the plan? Select all that apply.

  • A. To avoid activities that require bending over
  • B. To contact the surgeon if eye scratchiness occurs
  • C. To place an eye shield on the surgical eye at bedtime
  • D. That episodes of sudden severe pain in the eye is expected
  • E. To contact the surgeon if a decrease in visual acuity occurs
  • F. To take acetaminophen (Tylenol) for minor eye discomfort

Correct Answer: A, C, E, & F.

After the surgery is over, the client may be brought to a recovery room for a couple of hours before he will be allowed to go home. The client will be given prescriptions for antibiotics and anti-inflammatory drops to use at home as directed. He will be given an Implant Identification Card, which he should keep as a permanent record of the lens that was implanted in the eye.

  • Option A: Avoid activities that increase intraocular pressure such as bending over. Try not to bend from the waist to pick up objects on the floor, as this can cause undue pressure to the eyes. Do not lift any heavy objects.
  • Option B: After the surgical procedure, the client may be sensitive to light and have a feeling that something is in the eye. The client may experience minor discomfort after the procedure. The doctor may prescribe pain medication to make the client more comfortable during the first few days after the surgery.
  • Option C: The client is instructed to place an eye shield over the operative eye at bedtime to protect the eye from injury during sleep. The shield is meant to prevent rubbing of the eye or putting pressure on the eye while sleeping and to protect the eye from accidentally being hit or poked while it is healing.
  • Option D: The client should contact a doctor immediately if he develops severe pain or if his vision or other symptoms get worse instead of better. Follow all postoperative instructions given by the surgeon and surgical center.
  • Option E: The nurse would also instruct the client to notify the surgeon of purulent drainage, increased redness, or any decrease in visual acuity. Use the prescribed medications to help minimize the risk of infection and inflammation. Serious infection or inflammation can result in loss of vision.
  • Option F: After eye surgery, some scratchiness and mild eye discomfort may occur in the operative eye and is usually relieved by mild analgesics. If the eye pain becomes severe, the client should notify the surgeon because this may indicate hemorrhage, infection, or increased intraocular pressure. 

NURSESLABS-SATA-2-020

A nurse is teaching a client with left leg weakness to walk with a cane. The nurse should include which nursing points about safe cane use in the client teaching? Select all that apply.

  • A. Place the cane 8″ to 10″ from the base of the little toe.
  • B. Hold the cane on the uninvolved side of the body.
  • C. Adjust the cane so that the handle is level with the hip bone.
  • D. Walk by moving the involved leg, then the cane, and then the uninvolved leg.
  • E. Shorten the stride length on the involved side.
  • F. Avoid leaning on the cane to get in and out of a chair.

Correct Answer: B, C, & F.

A cane can be helpful if there are minor problems with balance or stability, some weakness in the leg or trunk, an injury, or a pain. If elderly, using a single point cane may help to walk more comfortably and safely and, in some cases, may make it easier to continue living independently.

  • Option A: The cane base should be placed 4″ to 6″ from the base of the little toe. The elbow should be slightly bent when holding the cane.
  • Option B: Hold the cane in the hand opposite the side that needs support. For example, if the left leg is injured, hold the cane in the right hand.
  • Option C: To ambulate safely, a client with leg weakness should hold the cane in the hand opposite the involved leg with the handle level adjacent to the hip bone. When standing up straight, the top of the cane should reach the crease in the wrist.
  • Option D: When walking, the client should move the cane and involved leg simultaneously, alternating with the uninvolved leg in equal length strides and timing.
  • Option E: To start, set the cane about one small stride ahead and step off on the injured leg. Finish the step with the good leg.
  • Option F: The client should not lean on the cane to get in or out of a chair because of the risk of falls.

NURSESLABS-SATA-2-021

A nurse is providing a list of instructions to a client who is scheduled to have an electroencephalogram (EEG). Choose the instructions that the nurse places on the list. Select all that apply.

  • A. Cola is acceptable to drink on the day of the test.
  • B. Tea and coffee are restricted on the day of the test.
  • C. The test will take between 45 minutes and 2 hours.
  • D. The hair should be washed the evening before the test.
  • E. All medications need to be withheld on the day of the test.
  • F. A nothing-by-mouth (NPO) status is required on the day of the test.

Correct Answer: B, C, & D.

Pre-procedure instructions include informing the client that the procedure is painless. An electroencephalogram (EEG) is an essential tool that studies the brain’s electrical activity. It is primarily used to assess seizures and conditions that may mimic seizures.

  • Options A and B: The procedure requires no dietary restrictions other than avoidance of cola, tea, and coffee on the morning of the test. These products have a stimulating effect and should be avoided. 
  • Option C: The client is informed that the test will take 45 minutes to 2 hours. Brain waves are typically recorded for 20-40 minutes after the leads are placed.
  • Option D: The hair should be washed the evening before the test, and gels, hairsprays, and lotion should be avoided. They are recommended not to use conditioner or other substances that might affect the quality of the recording (electrode impedance). The scalp is usually cleaned well to obtain proper recording with low impedance.
  • Options E and F: Medications are usually not withheld before the test. The client may eat, drink and take prescribed medications as usual.

NURSESLABS-SATA-2-022

The nurse is providing discharge teaching to the client who was given a prescription for nifedipine (Adalat) for blood pressure management. Which instructions should the nurse include? Select all that apply.

  • A. “Increase water intake.”
  • B. “Increase calcium intake.”
  • C. “Take pulse rate each day.”
  • D. “Weigh at the same time each day.”
  • E. “Palpitations may occur early in therapy.”
  • F. “Be careful when rising from sitting to standing.”

Correct Answers: C, D, E, & F.

Nifedipine is a calcium-channel blocker. Its therapeutic outcome is to decrease blood pressure. Its method of action is blockade of the calcium channels in vascular smooth muscle, promoting vasodilation. 

  • Option A: Increased water intake is not indicated in the client with cardiovascular disease. This medicine may cause fluid retention (edema) in some patients.
  • Option B: Nifedipine does not affect serum calcium levels. During the depolarization phase of smooth muscle cells, there is an influx of calcium ions through voltage-gated channels. Nifedipine inhibits the entry of calcium ions by blocking these voltage-dependent L-type calcium channels in vascular smooth muscle and myocardial cells.
  • Option C: Also, the client is taught to take his or her own pulse. Nifedipine is a calcium channel blocker. It works by affecting the movement of calcium into the cells of the heart and blood vessels. As a result, nifedipine relaxes blood vessels and increases the supply of blood and oxygen to the heart while reducing its workload.
  • Option D: Weight should be checked regularly to monitor for early signs of heart failure. Since nifedipine may potentially cause fluid build-up, it is also essential to check the client’s weight regularly.
  • Option E: Side effects that can occur early in therapy include reflex tachycardia (palpitations) and first-dose hypotension, leading to orthostatic hypotension.
  • Option F: Reduced intracellular calcium reduces peripheral arterial vascular resistance and dilatation of coronary arteries, leading to a reduction in systemic blood pressure and increased myocardial oxygen delivery. Nifedipine thus has hypotensive and antianginal properties.

NURSESLABS-SATA-2-023

A nurse is providing teaching regarding the prevention of Lyme disease to a group of teenagers going on a hike in a wooded area. Which of the following points should the nurse include in the session? Select all that apply.

  • A. Tuck pant legs into socks.
  • B. Wear closed shoes when hiking.
  • C. Apply insect repellent containing DEET.
  • D. Cover the ground with a blanket when sitting.
  • E. Remove attached ticks by grasping with thumb and forefinger.
  • F. Wear long sleeves and long pants in dark colors when in high-risk areas.

Correct Answer: A, B, C, & D.

Lyme disease or Lyme borreliosis is the most commonly transmitted tick-borne infection in the United States and among the most frequently diagnosed tick-borne infections worldwide.

  • Options A, B, and F: Measures to prevent tick bites focus on covering the body as completely as possible. Long sleeves and pants tucked into the socks along with closed shoes will offer some protection. Light-colored clothing should be worn so that ticks would be easily visible. 
  • Option C: Spraying insect repellent containing DEET on the skin and clothing can also prevent ticks. Permethrin can be used to treat boots, clothing and camping gear and remain protective through several washings.
  • Option D: Hikers should not sit directly on the ground and should cover the ground with an item such as a blanket. Ticks live in grassy, brushy, or wooded areas, or even on animals. Spending time outside walking the dog, camping, gardening, or hunting could bring in close contact with ticks.
  • Option E: Ticks should be removed with tweezers. Use clean, fine-tipped tweezers to grasp the tick as close to the skin’s surface as possible. Pull upward with steady, even pressure. Don’t twist or jerk the tick; this can cause the mouth-parts to break off and remain in the skin. If this happens, remove the mouth-parts with tweezers.

NURSESLABS-SATA-2-024

A nurse is reinforcing instructions to a client following a total laryngectomy about caring for the stoma. Choose the instructions that the nurse provides to the client. Select all that apply.

  • A. Protect the stoma from water.
  • B. Soaps should be avoided near the stoma.
  • C. Wash the stoma daily using a washcloth.
  • D. Use diluted alcohol on the stoma to clean it.
  • E. Apply a thin layer of petroleum jelly to the skin surrounding the stoma.
  • F. Use soft tissues to clean any secretions that accumulate around the stoma.

Correct Answer: A, B, C, & E.

An ostomy is a surgically created opening from the urinary tract or intestines, where effluent (fecal matter, urine, or mucous) is rerouted to the outside of the body using an artificially created opening called a stoma. A stoma typically protrudes above the skin, is pink to red in colour, moist, and round, with no nerve sensations.

  • Option A: The client is instructed to protect the stoma from water. Clean the skin around the stoma with water. Dry the skin completely before putting on the skin barrier or pouch.
  • Option B: Soap will not irritate it, but soap may interfere with the skin barrier sticking to the skin. It’s best to only use water while cleaning the skin around the stoma.
  • Option C: The client with a stoma should be instructed to wash the stoma daily with a washcloth. Normal exposure to air or contact with soap and water won’t harm the stoma. Water will not flow into the stoma.
  • Option D: The client should be instructed to avoid applying alcohol to a stoma because it is both drying and irritating. Do not use alcohol or any other harsh chemicals to clean the skin or stoma. They may irritate the skin. Do not use baby wipes or towelettes that contain lanolin or other oils, as these can interfere with the skin barrier adhesive and may irritate the skin.
  • Option E: A thin layer of petroleum jelly applied to the skin around the stoma helps prevent cracking. Large areas of skin that are red, sore, and weeping (always wet) will keep from getting a good seal around the stoma. It’s important to treat minor irritations right away.
  • Option F: Cotton swabs or tissues should be avoided because their particles may enter and obstruct the airway.

NURSESLABS-SATA-2-025

A nurse is reviewing the health records of assigned clients. The nurse plans care knowing that which client is at risk for fluid volume deficit? Select all that apply.

  • A. The client with cirrhosis
  • B. The client with a colostomy
  • C. The client with diarrhea
  • D. The client with third-degree burns
  • E. The client with decreased kidney function
  • F. The client with congestive heart failure (CHF)

Correct Answer B, C, & D.

Causes of a fluid volume deficit include vomiting, diarrhea, conditions that cause increased respirations or increased urinary output, insufficient intravenous fluid replacement, draining fistulas, ileostomy, and colostomy. A client with cirrhosis, CHF, or decreased kidney function is at risk for fluid volume excess.

  • Option A: Ascites is most often caused by liver scarring, otherwise known as cirrhosis. Scarring increases pressure inside the liver’s blood vessels. The increased pressure can force fluid into the abdominal cavity, resulting in ascites.
  • Option B: When the colon (large intestine) is removed, a greater risk for electrolyte imbalance can occur. The effluent characteristics of an ileostomy are between normal ileal and fecal content. There is fluid and electrolyte loss as the small bowel is unable to conserve sodium, chloride, and bicarbonate leading to dehydration, hyponatremia and metabolic acidosis.
  • Option C: All the acute effects of watery diarrhea result from the loss of water and electrolytes from the body in liquid stool. Additional amounts of water and electrolytes are lost when there is vomiting, and water losses are also increased by fever.
  • Option D: Severe fluid loss is the greatest problem faced following major burn injuries. Therefore, effective fluid resuscitation is one of the cornerstones of modern burn treatment.
  • Option E: Progressive loss of renal function causes reduced sodium filtration and inappropriate suppression of tubular reabsorption that ultimately lead to volume expansion. Fluid overload frequently manifests in patients with moderate to particularly late stages of CKD and has been associated with hypertension, congestive heart failure (CHF), left ventricular hypertrophy (LVH) as well as edema.
  • Option F: Heart failure can disturb the normal functioning of the kidney, weakening its ability to excrete sodium from the body and triggering mechanisms that cause water retention resulting in fluid overload.

NURSESLABS-SATA-2-026

A nurse is told in a report that a client has a positive Chvostek’s sign. What other data would the nurse expect to find on data collection? Select all that apply.

  • A. Coma
  • B. Tetany
  • C. Diarrhea
  • D. Possible seizure activity
  • E. Hypoactive bowel sounds
  • F. Positive Trousseau’s sign

Correct Answer: B, C, D, & F.

A positive Chvostek’s sign is indicative of hypocalcemia. Other signs and symptoms include tachycardia, hypotension, paresthesias, twitching, cramps, tetany, seizures, positive Trousseau’s sign, diarrhea, hyperactive bowel sounds, and a prolonged QT interval.

  • Option A: Severe hypercalcemia can damage the kidneys, limiting their ability to cleanse the blood and eliminate fluid. Severe hypercalcemia can lead to confusion, dementia and coma, which can be fatal.
  • Option B: Tetany is generally induced by a rapid decline in serum ionized calcium. Tetany is usually more dangerous and most commonly seen in the presence of respiratory alkalosis causing hypocalcemia.
  • Option C: The presence of chronic diarrhea or intestinal disease (eg, Crohn disease, sprue, chronic pancreatitis) suggests the possibility of hypocalcemia due to malabsorption of calcium and/or vitamin D.
  • Option D: Seizures are usually present in very severe hypocalcemia. It can be the sole manifestation or a part of the myriad of clinical presentations.
  • Option E: Signs of chronic hypocalcemia include hyperactive bowel sounds, dry and brittle hair, and abnormal clotting.
  • Option F: It represents increased neuromuscular excitability which may be related to the gating function of calcium ions for ion channels at a cellular level (particularly in neurons). It manifests as a spasm of the hand characterized by adduction of the thumb, flexion of the metacarpophalangeal joints, an extension of the interphalangeal joints, and flexion of the wrist when a sphygmomanometer is inflated above systolic blood pressure for three minutes.

NURSESLABS-SATA-2-027

A nurse lawyer provides an education session to the nursing staff regarding client rights. A nurse asks the lawyer to describe an example that may relate to invasion of client privacy. A nursing action that indicates a violation of this right is? Select all that apply.

  • A. Threatening to place a client in restraints
  • B. Performing a surgical procedure without consent
  • C. Taking photographs of the client without consent
  • D. Telling the client that he or she cannot leave the hospital
  • E. Turning computer screens showing off records towards the view of the public or people passing by

Correct Answer: C & E.

Invasion of privacy takes place when an individual’s private affairs are intruded on unreasonably. In health care, illicit (i.e., unauthorized) use of documentary materials related to treatment or condition of a patient is an invasion of privacy.

  • Option A:  Threatening to place a client in restraints constitutes assault. Assault is the intentional act of making someone fear that you will cause them harm. Threatening them verbally or pretending to hit them are both examples of assault that can occur.
  • Option B: Performing a surgical procedure without consent is an example of battery. Battery is the intentional act of causing physical harm to someone. Unlike assault, one doesn’t have to warn the victim or make him fearful before they are hurt for it to count as battery.
  • Option C: Photographs of patients are generally obtained within the doctor–patient relationship. Patients’ photographic/video records too are confidential documents and need to be respected similarly. Patients have autonomy over their bodies, and therefore, have the right to make their own decisions on medical treatment and allow subjecting themselves to photography/video filming.
  • Option D: Not allowing a client to leave the hospital constitutes false imprisonment. False imprisonment is the tort of restraining a person that person’s will. The person may be physically restrained (tied up or locked up), restrained by drugs, or restrained by threats. The recovery in a lawsuit based on false imprisonment includes damages for physical harm and psychological harm.
  • Option E: The client’s records should never be shown to the public. HIPAA allows that accidental disclosures may happen, but requires all covered entities — health plans, healthcare providers, and healthcare clearinghouses — to have “reasonable safeguards and minimum necessary policies and procedures” to prevent disclosing information whenever possible.

NURSESLABS-SATA-2-028

A nurse notes in the medical record that a client with Cushing’s syndrome is experiencing fluid overload. Which interventions should be included in the plan of care? Select all that apply.

  • A. Monitoring daily weight
  • B. Monitoring intake and output
  • C. Maintaining a low-potassium diet
  • D. Monitoring extremities for edema
  • E. Maintaining a low-sodium diet

Correct Answer: A, B, D, & E.

Cushing’s disease is a cluster of clinical abnormalities caused by excessive levels of adrenocortical hormones (particularly cortisol) or related corticosteroids and, to a lesser extent, androgens and aldosterone.

  • Option A: These changes may include fullness and rounding of the face (moon face), added fat on the back of the neck (buffalo hump), excessive weight gain, red cheeks (plethora), increased body and facial hair and Hyperpigmentation of skin, hair, and mucous membranes occurs as a result of the increased level of melanocyte-stimulating hormones and ACTH.
  • Option B: Cushing’s disease may result in increased blood pressure resulting from the expanded fluid volume with sodium and water retention. Tachycardia happens as a compensatory response to circulatory overload.
  • Option C: The client with Cushing’s syndrome experiencing fluid overload should be maintained on a high-potassium and low-sodium diet. Excessive cortisol causes sodium and water retention, edema, and increased potassium excretion. Mineralocorticoids regulate sodium and potassium secretion, and excess levels cause marked sodium and water retention as well as marked hypokalemia.
  • Option D: Detection of signs of circulatory overload will help in the immediate intervention. Due to excessive glucocorticoid and mineralocorticoid secretion, the client is predisposed to water and sodium retention.
  • Option E: Decreased sodium intake decreases renal retention of sodium and water. Too much sodium in the diet promotes fluid retention and weight gain. There should be adequate potassium in the diet since the elevation of cortisol level causes hypokalemia.

NURSESLABS-SATA-2-029

While providing care to a married client, the nurse notes multiple blue, purple, and yellow ecchymotic areas on the arms and trunk. When the nurse asks about these bruises, the client responds, “I tripped.” What actions should the nurse take? Select all that apply.

  • A. Document the client’s statement and complete a body map indicating the size, color, shape, location, and type of injuries.
  • B. Contact the local authorities to report suspicions of abuse.
  • C. Assist the client in developing a safety plan for times of increased violence.
  • D. Call the client’s spouse to arrange a meeting to discuss the situation.
  • E. Instruct the client to leave the abusive situation as soon as possible.
  • F. Provide the client with telephone numbers of local shelters and safe houses.

Correct Answer: A, C, & F.

Family and domestic violence is a common problem in the United States, affecting an estimated 10 million people every year; as many as one in four women and one in nine men are victims of domestic violence.

  • Option A: The nurse should objectively document the assessment findings. A detailed description of physical findings of abuse in the medical record is essential if legal action is pursued. 
  • Option B: The nurse should not report this suspicion of abuse because the client is a competent adult who has the right to self-determination. The healthcare provider needs to assure the patient that the decision is voluntary and that the provider will help regardless of the decision. The goal is to make resources accessible, safe, and enhance support.
  • Options C and F: All individuals, men or women, suspected of being abuse victims should be counseled on a safety plan, which consists of recognizing escalating violence within the family, formulating a plan to exit quickly, and knowing the telephone numbers of local shelters and safe houses. 
  • Option D: Contacting the client’s spouse without consent violates confidentiality. If the patient elects to leave their current situation, information for referral to a local domestic violence shelter to assist the victim should be given.
  • Option E: The nurse should respond to the client in a nonthreatening manner that promotes trust, rather than ordering the client to break off the relationship. During the initial assessment, a practitioner must be sensitive to the patient’s cultural beliefs. Incorporating a cultural sensitivity assessment with a history of being victims of domestic violence may allow more effective treatment.

NURSESLABS-SATA-2-030

Which instruction should the nurse provide to the client with diabetes mellitus receiving acarbose (Precose)? Select all that apply.

  • A. “Take the medication at bedtime.”
  • B. “Take the medication with each meal.”
  • C. “Take the medication on an empty stomach.”
  • D. “Side effects include abdominal bloating and flatus.”
  • E. “Take some form of glucose if hypoglycemia occurs.”
  • F. “Report symptoms such as shortness of breath or tiredness.”

Correct Answer: B, D, E, & F.

The mechanism of action of acarbose is a delay in absorption of dietary carbohydrates, thereby reducing the rise in blood glucose after a meal. Acarbose is FDA approved for the treatment of adults with type 2 diabetes mellitus as an adjunct to diet only or diet and exercise, depending on the patient’s health status.

  • Option A: Acarbose acts locally in the gastrointestinal (GI) tract with low systemic bioavailability (less than 2% gets absorbed as the active drug, and 35% as metabolites). Although the acarbose 300 mg three times daily regimen is superior to lower doses at lowering hemoglobin A1c, the approved maximum daily dose of acarbose is 100 mg three times daily.
  • Option B: To accomplish this, the medication must be taken with each meal. Acarbose is available as a 25 mg, 50 mg, or 100 mg oral tablet. It should be administered orally three times daily with the first bite of each meal. Initial dosing is 25 mg orally three times daily; however, starting with once-daily dosing may limit GI adverse effects.
  • Option C: From 25 mg by mouth three times daily, the dose can be titrated every 4 to 8 weeks to reach desired glycemic control while limiting GI adverse effects. The maximum daily dose is 100 mg three times daily.
  • Option D: Because of its bacterial fermentation of unabsorbed carbohydrates in the colon, side effects such as borborygmus, cramps, abdominal distention, and flatulence can occur. A high carbohydrate diet may worsen the GI adverse effects. GI symptoms tend to become reduced throughout treatment.
  • Option E: Hypoglycemia should not occur with acarbose monotherapy. However, the therapy can increase the risk of hypoglycemia when used with antidiabetic agents that cause hypoglycemia such as sulfonylureas or insulins.
  • Option F: The medication also can affect absorption of iron, leading to symptoms (shortness of breath, tiredness) of anemia.

NURSESLABS-SATA-2-031

A nurse prepares a list of home care instructions for the parents of a child who has a plaster cast applied to the left forearm. Choose the instructions that would be included on the list. Select all that apply.

  • A. Use the fingertips to lift the cast while it is drying.
  • B. Keep small toys and sharp objects away from the cast.
  • C. Use a padded ruler or another padded object to scratch the skin under the cast if it itches.
  • D. Place a heating pad on the lower end of the cast and over the fingers if the fingers feel cold.
  • E. Contact the health care provider if the child complains of numbness or tingling in the extremity.
  • F. Elevate the extremity on pillows for the first 24 to 48 hours after casting to prevent swelling.

Correct Answer: B, E, & F.

Casts and splints support and protect injured bones and soft tissue. Casts and splints hold the bones in place while they heal. They also reduce pain, swelling, and muscle spasm. In some cases, splints and casts are applied following surgery.

  • Option A: While the cast is drying, the palms of the hands are used to lift the cast. If the fingertips are used, indentations in the cast could occur and cause constant pressure on the underlying skin. 
  • Option B: Small toys and sharp objects are kept away from the cast. Keep dirt, sand, and powder away from the inside of the splint or cast. Inspect the skin around the cast. If the skin becomes red or raw around the cast, contact the doctor.
  • Option C: No objects (including padded objects) are placed inside of the cast because of the risk of altered skin integrity. Do not stick objects such as coat hangers inside the splint or cast to scratch itching skin. Do not apply powders or deodorants to itching skin.
  • Option D: A heating pad is not applied to the cast or fingers. Cold fingers could indicate neurovascular impairment, and the HCP should be notified. Numbness and tingling in the hand or foot may be caused by too much pressure on the nerves.
  • Option E: The extremity is elevated to prevent swelling, and the HCP is notified immediately if any signs of neurovascular impairment develop. Prop the injured arm or leg up above your heart by putting it on pillows or some other support. The client will have to recline if the splint or cast is on the leg. Elevation allows clear fluid and blood to drain “downhill” to the heart.

NURSESLABS-SATA-2-032

A nurse reinforces instructions to the mother of a child who has been hospitalized with a croup. Which of the following statements, if made by the mother, would indicate the need for further instruction? Select all that apply.

  • A. “I will give my child cough syrup if a cough develops.”
  • B. “During an attack, I will take my child to a cool location.”
  • C. “I will give acetaminophen (Tylenol) if my child develops a fever.”
  • D. “I will be sure that my child drinks at least three to four glasses of fluids every day.”
  • E. “I will place my child in a room with dry air.”
  • F. “I will let my child sit under the shower until the cough subsides.”

Correct Answer: A, E, & F.

Croup is a common respiratory illness of the trachea, larynx, and bronchi that can lead to inspiratory stridor and barking cough. The parainfluenza virus typically causes croup, but a bacterial infection can also cause it. Croup is primarily a clinical diagnosis.

  • Option A: Cough syrups and cold medicines are not to be given, because they may dry and thicken secretions. Cough medicines, which usually contain dextromethorphan or guaifenesin, are discouraged.
  • Option B: During a croup attack, the child can be taken to a cool basement or garage. Provide a calm, quiet environment for the child. Anxiety affects respirations and a calm environment lessens anxiety.
  • Option C: Acetaminophen is used if a fever develops. Educate the parents on the administration and uses of prescribed medications. This facilitates appropriate medication administration and recognition of adverse side effects.
  • Option D: Adequate hydration of 500 to 1000 mL of fluids daily is important for thinning secretions. Advise increase fluid intake and maintain intravenous fluid as prescribed. Adequate hydration can help loosen mucus in the oropharynx and prevent dehydration.
  • Option E: Have the child breathe moist air. Warm, moist air may help the child breathe easier. Cool mist and humidity soothe inflamed airways and decrease the viscosity of the mucus thus helps in clearing the airway.
  • Option F: If the child has symptoms of croup, take him into the bathroom, close the bathroom door, and turn on a hot shower. Do not put your child under the shower. Sit with the child in the warm, moist air for 15 to 20 minutes.

NURSESLABS-SATA-2-033

The nurse would anticipate the use of which medications in the treatment of the client with heart failure? Select all that apply.

  • A. Diuretics
  • B. Anticoagulants
  • C. Anticholinergics
  • D. Cardiac glycosides
  • E. Phosphodiesterase (PDE) inhibitors
  • F. Angiotensin-converting enzyme (ACE) inhibitors

Correct Answer: A, D, E, and F.

Medications recommended for treatment of heart failure include diuretics, cardiac glycosides such as digoxin (Lanoxin), PDE inhibitors, and ACE inhibitors. Clients in heart failure do not need anticoagulants or anticholinergics.

  • Option A: Pulmonary edema, most commonly resulting from HF, is also an indication for diuretic use. Loop diuretics (due to their greater effectiveness) are the cornerstone of diuretic therapy in symptomatic HF, with furosemide being the most widely used loop diuretic.
  • Option B: Anticoagulants derive their effect by acting at different sites of the coagulation cascade. Some act directly by enzyme inhibition, while others indirectly, by binding to antithrombin or by preventing their synthesis from the liver (vitamin K dependent factors).
  • Option C: Drugs with anticholinergic activity are useful for treating respiratory disorders (asthma, COPD), Parkinson’s, cardiovascular disease, urge incontinence, psychiatric disorders, depression, mydriasis, and allergies.
  • Option D: Cardiac glycosides, including digitalis and digoxin, have long-standing use in clinical practice. This drug received approval from the FDA in 1954 and is used to treat various heart problems such as atrial flutter, atrial fibrillation, heart failure with its associated symptoms and to induce fetal demise prior to an abortion.
  • Option E: Phosphodiesterase inhibitors are class medications used in the management and treatment of chronic obstructive pulmonary disease(COPD), erectile dysfunction(ED), pulmonary arterial hypertension(PAH), benign prostatic hyperplasia(BPH), acute decompensated heart failure, psoriasis, psoriatic arthritis(PA), atopic dermatitis(AD), and neonatal apnea.
  • Option F: Angiotensin-converting enzyme (ACE) inhibitors are useful as adjunctive therapy in systolic heart failure (HF). HF guidelines recommend ACE inhibitors to help prevent HF in patients with a reduced ejection fraction (EF) who also have a history of myocardial infarction (MI), and to prevent HF in any patient with a reduced ejection fraction (EF), or to treat patients with HF, and reduced EF.

NURSESLABS-SATA-2-034

The nurse is providing care for a client who has had a stroke. Since the onset of symptoms, the client has been experiencing left-sided hemianopsia. Which nursing interventions would be appropriate? Select all that apply.

  • A. Place the client’s belongings on the right side of the bed.
  • B. Approach the client from the left side. 
  • C. Refuse to acknowledge the condition to promote the client’s independence.
  • D. Stand on the right side of the bed when providing care.
  • E. Provide the client with an eye patch for the right eye.
  • F. Dim the lights in the room to prevent eye strain.

Correct Answer: A & D.

Hemianopsia is a condition in which the client has lost half of the visual field. It is most often associated with stroke. Homonymous hemianopsia may result from stroke, head trauma, mass occupying lesions, invasive surgical procedures, or neurologic conditions such as multiple sclerosis, Alzheimer’s disease, and epilepsy.

  • Option A: In this case, the stroke has affected the client’s left side; therefore, placing belongings on the right side of the bed will enable the client to best see them. In order to improve patients’ ability to compensate for their visual loss, several researchers have developed training schemes designed to teach patients more efficient strategies for visual scanning.
  • Option B: Approaching the client from the left side is counterproductive because the client would not be able to adequately see the nurse. The compensatory training approaches typically use target-localization tasks to train patients to make large eye-movements and use visual search tasks to teach patients to use systematic scanning strategies when searching their visual world.
  • Option C: Due to the enjoyment which can be gained from reading and other leisure activities requiring visual search skills, any impairment in these has obvious consequences for the emotional well-being of the patient.
  • Option D: Standing on the right side of the bed when providing care will ensure the client is able to see the nurse. Optical aids such as prism glasses can be used to reduce the apparent visual field loss by shifting visual stimuli from the blind field into the patient’s seeing field. These prisms are fitted to spectacles but need to be restricted to just one half of each of the lenses (typically on the side of the blind field).
  • Option E: Using an eye patch will not help with treating or managing the condition.  Patient recovery may benefit from a multifaceted approach that includes visual training, visual assist devices (prism correction), occupational therapy, and psychological rehabilitation.
  • Option F: Dimming the lights would further decrease seeing the visual field. Some patients can respond quite accurately to visual stimuli presented to their blind field (for example by pointing to it) even though they insist that they cannot see it. This phenomenon has been called blindsight.

NURSESLABS-SATA-2-035

A nurse is caring for a client who is disoriented to time, place, and person and is attempting to get out of bed and pull out an intravenous (I.V.) line that is supplying hydration and antibiotics. The client has a vest restraint and bilateral soft wrist restraints. Which nursing actions would be appropriate? Select all that apply.

  • A. Perform a face-to-face behavior evaluation every hour.
  • B. Tie the restraints in quick-release knots.
  • C. Tie the restraints to the side rails of the bed.
  • D. Document the client’s condition.
  • E. Document alternative methods used before the restraints were applied.
  • F. Document the client’s response to the intervention.

Correct Answer: A, B, D, E, & F.

Preventing a client from falling or harming him- or herself is of utmost importance. Applying restraints should be a last resort when all other alternative interventions have been attempted. 

  • Option A: A face-to-face evaluation must be performed every hour. After restraint placement, patients should be reevaluated every hour and moved at regular intervals to prevent sequelae such as pressure ulcers, rhabdomyolysis, and paresthesias.
  • Option B: Restraints should be tied in knots that can be released quickly and easily. Physical restraints encompass hand mitts, soft cloth limb restraints, leather limb restraints, enclosed beds, belts, and vests. 
  • Option C: Restraints should never be secured to side rails because doing so can cause injury if the side rail is lowered without untying the restraint. Ideally, a restraint team should include at least five people, including the team leader.
  • Options D, E, and F: The nurse should document the client’s condition, any alternative methods used before the restraints were applied, and the client’s response to the interventions. Document appropriate clinical indication and have a standardized checklist prepared for staff to monitor and supply patient needs effectively.

NURSESLABS-SATA-2-036

Which of these clients are most likely to develop fluid (circulatory) overload? Select all that apply.

  • A. A premature infant
  • B. A 101-year-old man
  • C. A client on renal dialysis
  • D. A client with diabetes mellitus
  • E. A 29-year-old woman with pneumonia
  • F. A client with congestive heart failure

Correct Answer: A, B, C, & F.

Clients with cardiac, respiratory, renal, or liver diseases and older and very young clients cannot tolerate an excessive fluid volume. The risk of fluid (circulatory) overload exists with these clients.

  • Option A: The preterm fetus or neonate is in a state of relative total body water and extracellular fluid excess. After birth this excess water must be mobilized and excreted. A proportion of the diuresis observed in both term and preterm infants during the first days of life should be regarded as physiologic.
  • Option B: The elderly population is also at risk for overhydration. Overhydration, or fluid overload, is caused by a number of conditions, including heart failure, kidney failure, and protein deficiency. Iatrogenic overhydration can also occur as a result of receiving intravenous fluids, blood transfusions, and steroids.
  • Option C: Achieving a balance between avoiding hypovolemia during dialysis and developing fluid overload between dialysis sessions is complicated by patient adherence, challenges in assessing fluid status, and limitations on the length of dialysis sessions.
  • Option D: When there is diabetes, excess glucose — a type of sugar — builds up in the blood. The kidneys are forced to work overtime to filter and absorb the excess glucose. When the kidneys can’t keep up, the excess glucose is excreted into the urine, dragging along fluids from the tissues, which makes the client dehydrated.
  • Option E: Advice to increase fluid intake is a frequent treatment recommendation. Attributed benefits of fluids include replacing increased insensible fluid losses, correcting dehydration from reduced intake and reducing the viscosity of mucus.
  • Option F: The body’s response to heart failure causes sodium levels to increase. To restore balance, the body retains water, leading to fluid overload and an increased burden on the heart.

NURSESLABS-SATA-2-037

A client has just been diagnosed with terminal cancer and is being transferred to home hospice care. The client’s daughter tells the nurse, “I don’t know what to say to my mother if she asks me if she’s going to die.” Which responses by the nurse would be appropriate? Select all that apply.

  • A. “Tell your mother not to worry. She still has some time left.”
  • B. “Let’s talk about your mother’s illness and how it will progress.”
  • C. “You sound like you have some questions about your mother dying. Let’s talk about that.”
  • D. “Don’t worry, hospice will take care of your mother.”
  • E. “Tell me how you’re feeling about your mother dying.”

Correct Answer: B, C, & E.

Talking about death is an uncomfortable situation. Conveying information clearly and openly can alleviate fears and strengthen the individual’s sense of control. Encouraging verbalization of feelings helps build a therapeutic relationship based on trust and reduces anxiety. Advising the daughter not to worry, or having her tell her mother that, ignores her feelings and discourages further communication.

  • Option A: The nurse needs to recognize and understand these events as a time during which an individual or family member incorporates his or her strength to go on to the next stage of grief.
  • Option B: Support the client and significant others share mutual fears, concerns, plans, and hopes for each other. Keeping secrets won’t do any help during this time. These times of stress can be used as an opportunity for growth and family development.
  • Option C: Communicate therapeutically with the client and family members and allow them to verbalize feelings. Sharing feelings with a healthcare provider may help the client and significant others find significance in the experience of loss.
  • Option D: Acknowledge the client’s and significant other’s need to review the loss experience. In this way, the client and family members integrate the event into their experience.
  • Option E: Review and point out strengths and progress to date. Reviewing the client’s progress is very helpful and provides perspective in the whole process.

NURSESLABS-SATA-2-038

When caring for a 3-year-old child, the nurse should provide which toys for this child? Select all that apply.

  • A. A puzzle
  • B. A wagon
  • C. A golf set
  • D. A farm set
  • E. A doll
  • F. A lightweight ball

Correct Answer: B, E, & F.

Toys for the toddler must be strong, safe, and too large to swallow or place in the ear or nose. Toddlers need supervision at all times. Push-pull toys, large balls, large crayons, trucks, and dolls are some appropriate toys. 

  • Option A: A puzzle with large pieces only may be appropriate. Wood puzzles with only 4 to 12 large pieces aid in the toddler’s development of critical thinking. Other appropriate toys may be blocks that snap together, objects to sort, and things with hooks, buttons, buckles and snaps.
  • Options C and D: A farm set and a golf set may contain items that the child could swallow. These kinds of toys are appropriate for preschoolers who like pretending and building. Preschoolers have a longer attention span than toddlers.
  • Options B and F: Toys that make the use of large and small muscles are also appropriate for toddlers to develop their gross motor skills. This may include large or small balls for kicking and throwing, ride-on equipment, push and pull toys such as wagons, and low climbers with soft material underneath.
  • Option E: A doll with accessories will catch the attention of a toddler too, as well as child-sized furniture, dress-up clothes, puppets, and sand and water play toys.

NURSESLABS-SATA-2-039

When the nurse is collecting data from the older adult, which of the following findings would be considered normal physiological changes? Select all that apply.

  • A. Increased heart rate
  • B. Decline in visual acuity
  • C. Decreased respiratory rate
  • D. Decline in long-term memory
  • E. Increased susceptibility to urinary tract infections
  • F. Increased incidence of awakening after sleep onset

Correct Answer: B, E, & F.

These changes with age have important practical implications for the clinical management of elderly patients: metabolism is altered, changes in response to commonly used drugs make different drug dosages necessary and there is a need for rational preventive programs of diet and exercise in an effort to delay or reverse some of these changes.

  • Option A:  The heart rate decreases and the heart valves thicken. The cardiac output decreases, blood pressure increases, and arteriosclerosis develops. An older heart cannot speed up as quickly or pump as fast or as much blood as a younger heart.
  • Option B: Anatomical changes to the eye affect the individual’s visual ability, which leads to potential problems with activities of daily living. Light adaptation and visual fields are reduced. Aging includes a decline in accommodation (presbyopia), glare tolerance, adaptation, low-contrast activity, attentional visual fields, and color discrimination.
  • Option C: Respiratory rates are usually unchanged. The lungs show impaired gas exchange, a decrease in vital capacity, and slower expiratory flow rates. The muscles used in breathing, the diaphragm, and muscles between the ribs, tend to weaken. The number of air sacs (alveoli) and capillaries in the lungs decreases.
  • Option D: Short-term memory may decline with age, but long-term memory is usually maintained. During normal aging, blood flow in the brain decreases and gets less efficient at recruiting different areas into operations. The whole group of changes taking place in the brain with agiing decreases the efficiency of cell-to-cell communication, which declines the ability to retrieve and learn
  • Option E: Age-related changes that affect the urinary tract increase an older client’s susceptibility to urinary tract infections. The muscle that controls the passage of urine out of the body (urinary sphincter) is less able to close tightly and prevent leakage. Thus, older people have more difficulty postponing urination.
  • Option F: Changes in sleep patterns are consistent, age-related changes. Older persons experience an increased incidence of awakening after sleep onset. Because older people sleep more lightly and wake up more often, they may feel deprived of sleep even when their total sleep time has not changed.

Questions and rationale from Nurseslabs.com Feel free to print or share and link back to us! For more practice questions, please visit our Nursing Test Bank [https://nurseslabs.com/nursing-test-bank]

NURSESLABS-SATA-2-040

Which data indicates to the nurse that a client may be experiencing ineffective coping? Select all that apply.

  • A. Constantly neglects personal grooming
  • B. Visits her husband’s grave once a month
  • C. Visits the senior citizens’ center once a month
  • D. Frequently looks at snapshots of her husband and family
  • E. Criticizing one’s self (negative self-talk)

Correct Answer: A & E.

Coping mechanisms are behaviors that are used to decrease stress and anxiety. In response to a death, ineffective coping is manifested by an extreme behavior that in some instances may be harmful to the individual, physically, psychologically, or both. 

  • Option A: This is indicative of a behavior that identifies an ineffective coping behavior as part of the grieving process. Patients using maladaptive coping mechanisms are more likely to engage in health-risk behaviors than those with appropriate mechanisms.
  • Option B: Emotion-focused, which aims to reduce the negative emotions associated with the problem: Examples of this style include positive reframing, acceptance, turning to religion, and humor.
  • Option C: Social coping (support-seeking) in which an individual reduces stress by seeking emotional or instrumental support from their community. 
  • Option D: Meaning-focused, in which an individual uses cognitive strategies to derive and manage the meaning of the situation.
  • Option E: The relation between maladaptive coping mechanisms and numerous disorders has been established. Psychiatric disorders such as PTSD, anxiety, and major depression, and somatic symptoms were all correlated with coping styles related to avoidance.

NURSESLABS-SATA-2-041

The nurse notes that a client is quite suspicious during an assessment interview and believes that her family is under investigation by the CIA. What would the appropriate nursing interventions be with this client? Select all that apply:

  • A. Use active listening skills to seek information from the client.
  • B. Encourage the client to describe the problem as she sees it.
  • C. Ask the client to tell you exactly what she thinks is happening.
  • D. Tell the client that she is delusional and you can help her.
  • E. Explain to the client that most people are not investigated by the CIA.
  • F. Reassure the client that you are not with the CIA.

Correct Answer: A, B, and C.

The client is displaying paranoid behaviors, which necessitates a matter of fact approach that is nonjudgmental and accepting the client’s statements and shows the nurses willingness to actively listen. The last three do not contribute to a therapeutic nurse client relationship.

  • Option A: Explore how the delusions are experienced by the client. Exploring the delusions and sharing the experience can help give the person a sense of power that he or she might be able to manage the delusions.
  • Option B: Help the client to identify times that the delusions are most prevalent and frightening. This helps both nurse and client identify situations and times that might be most anxiety-producing and threatening to the client.
  • Option C: Help the client to identify the needs that might underlie the delusions. What other ways can these needs be met? Delusions might reflect needs for anger, power, self-esteem, and sexuality.
  • Option D: Accept the fact that the paranoia is real to the client, but explain that you do not see his paranoia. Validating that your reality does not include delusions can help the client cast “doubt” on the validity of his or her thoughts.
  • Option E: Attempt to understand the significance of these beliefs to the client at the time of their presentation. Important clues to underlying fears and issues can be found in the client’s seemingly illogical fantasies.
  • Option F: Recognizes the client’s delusions as the client’s perception of the environment. Recognizing the client’s perception can help you understand the feelings he or she is experiencing.

NURSESLABS-SATA-2-042

Which nursing interventions will assist in reducing pressure points that may lead to pressure ulcers? Select all that apply:

  • A. Position the client directly on the trochanter when side-lying.
  • B. Avoid use of donut type devices.
  • C. Massage bony prominences.
  • D. Elevate the HOB no more than 30 degrees when possible.
  • E. When the client is side-lying, use the 30-degree lateral inclined position.
  • F. Avoid uninterrupted sitting in a chair or wheelchair.

Correct Answer: B, D, E, and F.

Prevention requires an on-going risk assessment, consideration of causal factors, implementation of prevention strategies and the selection of an appropriate use of pressure relieving devices. When an assessment identifies a patient at risk of pressure injury, interventions should be implemented immediately.

  • Option A: Do not place the client on their trochanter. Positions may include: prone, seated in bed, seated in chair, left side lying, right side lying and supine. Always check the positioning of the bony prominences and heels when repositioning the patient into any position.
  • Option B: Avoid donuts that promote ischemia. Encourage the use of pillows, foam wedges, and pressure-reducing devices to prevent pressure injury. Provide the right positioning aids and use of appropriate support surfaces to help reduce friction and shear. 
  • Option C: Don’t massage bony prominences as this causes capillary break down and injury leading to pressure ulcers. Do not vigorously rub or massage the clients’ skin. Use a pH appropriate skin cleanser and dry thoroughly to protect the skin from excess moisture.
  • Option D: Elevating the head of the bed to 30 degrees or less will decrease the chance of ulcer development from shearing forces. Maintain the head of the bed at the lowest degree of elevation possible to reduce shear and friction.
  • Option E: When placing the client in a side-lying position, use the 30-degree lateral inclined position. For patients who are unable to assist moving themselves, it is recommended that they be repositioned every two hours. Heels should be suspended off the bed using pillows or gel pads.
  • Option F: Consider smaller more frequent shifts in position of patients who cannot tolerate major changes in body position to redistribute pressure. For patients who are unable to assist moving themselves, it is recommended that they be repositioned every two hours.

NURSESLABS-SATA-2-043

The nurse is evaluating a client recently diagnosed with primary open angle glaucoma (POAG). What will an important nursing action be? Select all that apply.

  • A. Review meds the client is currently on to determine whether any of them cause an increased intraocular pressure as a side effect.
  • B. Determine whether the client has any sudden loss of vision accompanied by pain.
  • C. Discuss with the client the importance of controlling blood pressure to decrease the potential loss of peripheral vision.
  • D. Instruct the client to take analgesics as soon as any discomfort occurs in the eye and to notify the clinic if pain is not relieved.
  • E. Have the client demonstrate the use of eye drops.
  • F. Assess the client for chronic diseases such as diabetes.

Correct Answer: A, E, and F.

Open-angle glaucoma is a chronic, progressive, and irreversible multifactorial optic neuropathy that is characterized by an open angle of the anterior chamber, optic nerve head changes, progressive loss of peripheral vision, followed by central visual field loss.

  • Option A: Medications must be evaluated in terms of their potential for increasing the intraocular pressure. An increase in intraocular pressure could cause further damage to a patient with POAG.
  • Option B: POAG is painless. Early changes in OAG involve a loss of peripheral vision that the patient is usually not aware of until 40% of their nerve fibers have been compromised, only then do they start to notice having “tunnel vision.”
  • Option C: The question states the client is already diagnosed. Open-angle glaucoma is often asymptomatic in its early stages, therefore, a thorough and comprehensive history and exam can be instrumental in detecting the disease early.
  • Option D: POAG is not correlated to BP. Elevated intraocular pressure is an important risk factor for open-angle glaucoma and can be a result of primary or secondary causes.
  • Option E: Ophthalmic drops are often prescribed for glaucoma and clients should know how to administer them correctly. Some patients will attempt to use their drops every day but will fail to properly deliver the medications into their eyes and thus the medication will not be absorbed, specifically at-risk elder populations, who may struggle with administering drops into their own eyes.
  • Option F: Diabetes is a risk factor and its management is important in helping slow POAG. Type 2 diabetes is a risk factor for primary open-angle glaucoma. This has been demonstrated by large epidemiologic studies including the Los Angeles Latino Eye Study and the Blue Mountains eye study in Australia.

NURSESLABS-SATA-2-044

A nurse understands that a patient may experience pain during peritoneal dialysis because of which of the following? Select all that apply.

  • A. Warming the dialysate
  • B. Too rapid installation
  • C. Infiltration of the solution into the bloodstream
  • D. Accumulation of dialysate solution under the diaphragm
  • E. Too rapid outflow of the dialysate

Correct Answer: B and D.

Infusion pain is a frequent problem in peritoneal dialysis (PD), and can markedly vary in intensity and risk. In general, treatment of infusion pain is dictated by the specific cause.

  • Option A: Warming helps with discomfort. Warming the solution increases the rate of urea removal by dilating peritoneal vessels. Cold dialysate causes vasoconstriction, which can cause discomfort and excessively lower the core body temperature, precipitating cardiac arrest.
  • Option B: If pain is caused by jetting of dialysate against the peritoneum, reducing the infusion rate may alleviate the pain.
  • Option C: The dialysate does not infiltrate the circulation. Inadvertent introduction of air into the abdomen irritates the diaphragm and results in referred pain to the shoulder blade. This type of discomfort may also be reported during initiation of therapy or during infusions and usually is related to stretching and irritation of the diaphragm with abdominal distension.
  • Option D: Not only the microorganisms but also free air can enter the peritoneal cavity. Presence of free air in the peritoneal cavity is called pneumoperitoneum (PP). Abdominal pain in a patient with a PD catheter in situ has many potential differential diagnoses.
  • Option E: Rapid outflow doesn’t cause pain. Abdominal pain or discomfort during complete drain of dialysis solution or during installation of peritoneal dialysis solution into an empty peritoneal cavity is an infrequent finding in CPD patients. Usually the pain is noted during the drain phase, near the end of the drain.

NURSESLABS-SATA-2-045

The nurse is evaluating a client’s response to hemodialysis. Which lab results will indicate the dialysis was effective? Select all that apply.

  • A. Serum potassium level decreases from 5.4 to 4.6 mEq/L
  • B. Cr decreases from 1.6 to 0.8 mg/dL
  • C. Hgb increases from 10-12 g/dL
  • D. WBC increase from 5000 to 8000/mm3
  • E. BUN decreases from 110 to 90 mg/dL

Correct Answer: A, B, and E.

Primary action of hemodialysis is to clear nitrogenous waste products. Dialysis adequacy measures the effectiveness of the dialysis treatments. It is important to receive enough dialysis to feel well and minimize the side effects of kidney failure.

  • Option A: When the kidneys do not work properly, potassium may build up in the blood. A potassium level that is too high or too low may weaken muscles and change the heart rhythm. The ideal range for potassium in a person on dialysis is 3.5-5.5.
  • Option B: Dialysis has a positive impact on serum creatinine level and reduces its level towards normal value. Results showed that most of the patients (58%) had serum creatinine below 7 mg/dl after dialysis.
  • Option C: The hemoglobin is frequently low in people with kidney failure because the kidneys no longer make the hormone erythropoietin. This hormone stimulates the bones to make red blood cells. The ideal level for hemoglobin should be around 10g/dl. Recent research has demonstrated that levels above 13g/dl may be harmful to patients on dialysis.
  • Option D: Direct contact of the blood with the dialysis membrane during hemodialysis elicits a series of changes in blood cells. White blood cell count and total lymphocyte number are reduced, neutrocytes are stimulated and degranulate, platelet adhesiveness is enhanced.
  • Option E: The BUN is a measurement of waste products in the blood. Normal values for a person with kidney failure vary according to protein intake. A client’s values may range from 20-80 mg/dl when the labs are drawn prior to the dialysis treatment.

NURSESLABS-SATA-2-046

The nurse understands that the following clinical findings are indications for dialysis. Select all that apply.

  • A. Volume overload
  • B. BUN 18 mg/dL
  • C. K 5.2 mEq/L
  • D. Decreased creatinine clearance.
  • E. Metabolic acidosis
  • F. Cr 5.0 mg/dL

Correct Answer: A, C, E, and F.

Indications for dialysis include volume overload, weight gain, hyperkalemia, metabolic acidosis, and rising BUN (normally 10-20 mg/dL) and Cr (normally 0.5-1.5 mg/dL) levels, along with decreased urinary creatinine clearance. 

  • Option A: Hemodialysis initiation is needed for acute illness associated with AKI, life-threatening hyperkalemia, refractory acidosis, hypervolemia causing end-organ complications (e.g., pulmonary edema), or any toxic ingestion. Patients with CKD and heart failure experience fluid retention, which leads to worsening of heart failure and pulmonary edema.
  • Option B: The BUN is normal. The decision to initiate dialysis should not be based on the level of kidney function in an asymptomatic individual. In renal failure patients, elevated urea levels can also lead to uremic pericarditis.
  • Option C: The potassium level is hyperkalemic. The decision to initiate maintenance dialysis should be based on an assessment of signs and symptoms of renal failure (pruritus, acid-base or electrolyte abnormalities, serositis), volume or BP dysregulation, a progressive deterioration in nutritional status despite dietary intervention or impairment in cognition.
  • Option D: It is not advisable to assign an arbitrary urea nitrogen or creatinine level for dialysis initiation due to individual variability in uremia symptom severity and renal function.
  • Option E: Potassium abnormalities arise from acidosis (due to intracellular shift) and decreased renal excretion in chronic kidney disease or renal failure patients.
  • Option F: The National Kidney Foundation’s Kidney Disease Outcomes Quality Initiative (KDOQI) recommends that patients who reach CKD stage 4 (GFR, 30 mL/min/1.73 m^2), and those with an imminent need for maintenance dialysis during the initial assessment, should be counseled about renal failure and the treatment options (kidney transplantation, hemodialysis at home or in-center, PD) and conservative treatment.

NURSESLABS-SATA-2-047

The nurse is assessing a client who had a fractured femur repaired with an external fixator device. Which assessment finding would cause the nurse concern regarding the development of compartment syndrome? Select all that apply.

  • A. Decrease in pulse rate in affected leg.
  • B. Paresthesia distal to the area of injury.
  • C. Toes on affected leg are cool to touch and edematous.
  • D. Complaints that pins are hurting.
  • E. Complaints of leg pain unrelieved by analgesics or repositioning.
  • F. Client angry and calling loudly to the nurse every ten minutes.

Correct Answer: B, C, and E.

Paresthesia, edema, and leg pain unrelieved by analgesics are classic indicators of the development of compartment syndrome. Acute compartment syndrome typically occurs within a few hours of inciting trauma. However, it can present up to 48 hours after.

  • Option A: With a femur fracture, there will be a decrease in pulse strength. Beware that the presence or absence of a palpable arterial pulse may not accurately indicate relative tissue pressure or predict the risk for compartment syndrome. In some patients, a pulse is still present, even in a severely compromised extremity. 
  • Option B: Paresthesia, hypoesthesia, or poorly localized deep muscular pain may also be present. Classically, the presentation of acute compartment syndrome has been remembered by “The Five P’s”: pain, pulselessness, paresthesia, paralysis, and pallor. However, aside from paresthesia, which may occur earlier in the course of the condition, these are typically late findings.
  • Option C: Acute compartment syndrome occurs when there is increased pressure within a closed osteofascial compartment, resulting in impaired local circulation. The reduction of venous outflow and arterial inflow result in decreased oxygenation of tissues causing ischemia. If the deficit of oxygenation becomes high enough, irreversible necrosis may occur.
  • Option D: The pins do not usually cause pain, but this may be a sign of infection. Postoperatively, the pins are sometimes wrapped with xeroform or iodine impregnated gauze. Motion around the skin-pin junction is known to increase the risk of infection.
  • Option E: Pain is typically severe, out of proportion to the injury. Early on, pain may only be present with passive stretching. However, this symptom may be absent in advanced acute compartment syndrome.
  • Option F: Anger can be due to immobility. Acute compartment syndrome occurs more commonly in males younger than 35, which may be due to a larger relative intracompartmental muscle mass and increased likelihood of being involved in high-energy trauma.

NURSESLABS-SATA-2-048

The nurse is preparing a discharge for a patient with GERD. What would be important for the nurse to include in this teaching plan? Select all that apply.

  • A. Elevate the HOB.
  • B. Decrease intake of caffeine.
  • C. Discuss strategies for weight loss if overweight.
  • D. Increase fluid intake with meals.
  • E. Take ranitidine (Zantac) at hs.
  • F. Eat a bedtime snack of milk and protein.

Correct Answer: A, B, C, and E.

Gastroesophageal reflux disease or simply GERD is the excessive backflow of gastric or duodenal contents, or both into the esophagus and past the lower esophageal sphincter (LES) for a sustained length of time without associated belching or vomiting. This will all help neutralize stomach acid. 

  • Option A: Instruct to remain in upright position at least 1–2 hours after meals, and to avoid eating within 2–4 hours of bedtime. Gravity helps control reflux and causes less irritation from reflux action into the esophagus.
  • Option B: Instruct client regarding avoidance of alcohol, smoking, and caffeinated beverages. This increases acid production and may cause esophageal spasms.
  • Option C: Identify the amount of weight loss needed for optimal body size and frame. Establish a dietary plan for weekly goals of weight loss of one pound. Encourage the client to make gradual changes in dietary habits.
  • Option D: Drinking lots with meals and eating before bed will exacerbate the problem. Avoid an increase in fluid intake during meals because this can precipitate regurgitation.
  • Option E: Antacids and H2 receptor antagonists like famotidine (Pepcid), nizatidine (Axid), or ranitidine (Zantac) act by neutralizing the acid in the stomach, therefore, helps relieve pain.
  • Option F: Instruct the client to avoid highly seasoned food, acidic juices, alcoholic drinks, bedtime snacks, and foods high in fat. These can reduce the lower esophageal sphincter pressure.

NURSESLABS-SATA-2-049

The nurse is preparing a client for cardiac catheterization. Which nursing interventions are necessary in preparing the client for this procedure? Select all that apply.

  • A. Verify consent has been signed.
  • B. Explain procedure to client.
  • C. Provide clear liquid, no caffeine diet.
  • D. Evaluate peripheral pulses.
  • E. Obtain a 12 lead ECG
  • F. Obtain history of shellfish allergy.

Correct Answer: A, B, E, and F.

In cardiac catheterization, a contrast dye is injected into the coronary artery and provides info on patency. The term cardiac catheterization can refer to either right heart catheterization or left heart catheterization, or both. The procedure can be either diagnostic or therapeutic, and interventional cardiologists can perform a variety of interventions depending on the clinical need.

  • Option A: Informed consent must be signed prior to any invasive procedure. Most of the procedures can be performed with minimal or moderate sedation with the help of a local anesthetic, but some procedures will require anesthesia services for providing deep sedation or general anesthesia.
  • Option B: The physician is responsible for explaining the procedure, the nurse can reinforce. Preparation for the cardiac catheterization procedure starts with a thorough history of the patient along with a detailed examination. After defining the clinical question, the performing interventional cardiologist will decide on the access for the procedure.
  • Option C: The patient would be NPO 6-18 hours prior. The rationale of NPO in the setting of cardiac catheterization is to reduce the risk of aspiration, and more so, of a patient needing emergent cardiac surgery.
  • Option D: Peripheral pulses are important afterward. The most frequent complication after transradial access is about a 5% risk of radial artery occlusion. This is a clinically insignificant complication if the Allen test is normal. Patients with incomplete palmar arch and abnormal Allen test may have symptoms of hand ischemia after radial artery occlusion.
  • Option E: An ECG would be done but measures electrical, not blood flow. Basic workup includes a complete blood count (CBC), basic metabolic panel (BMP), prothrombin time, electrocardiogram, and chest X-ray
  • Option F: Shellfish is an indicator of an allergy to the medium injected. Patients with documented allergy to radio-iodinated contrast material will need premedication with corticosteroids and antihistamines.

NURSESLABS-SATA-2-050

The nurse has been assigned a group of cardiac clients. What would be the most important information for the nurse to check on the initial evaluation of each client? Select all that apply.

  • A. Presence of cardiac pain.
  • B. Medications taken before hospitalizations.
  • C. Presence of jugular vein distention.
  • D. Heart sounds and apical rate.
  • E. Presence of diaphoresis.
  • F. History of difficulty breathing.

Correct Answer: A, C, D, and E.

A focussed cardiac assessment is directed towards assessing physiological symptoms (cardiac pain, JVD, heart sounds and rate, and presence of diaphoresis) that provide immediate information regarding the client’s condition, which is appropriate for the nurse to do at the beginning of each shift. After the physiological parameters have been evaluated the nurse can determine history of SOB and meds.

  • Option A: The hallmark of cardiac disease, however, it can also result from pulmonary, GI, and musculoskeletal disorders. A detailed history is imperative to differentiate cardiac anginal chest pain from the rest. The typical description of angina is retrosternal and diffuse.
  • Option B: A thorough history and review of systems can help diagnose specific cardiac diseases and can help differentiate it from diseases of other organ systems.
  • Option C: Abnormalities in the jugular venous pulse can lead clinicians towards diseases such as atrial fibrillation, tricuspid regurgitation, tricuspid stenosis, pulmonary artery hypertension, pulmonic stenosis, and cardiac tamponade. Looking for jugular venous distention and performing the hepatojugular reflex can give a rough estimation of right-sided heart pressures.
  • Option D: Palpitations is defined as a fluttering, uncomfortable sensation in the chest that is usually associated with arrhythmias. It feels as if the “heart has skipped a beat.” When palpating the arterial pulse, the examiner should be able to gather the rate, rhythm, and characteristics.
  • Option E: The skin can display many clues about cardiac disease. Inspect the temperature of the skin, cool skin points towards poor perfusion, which is many times due to reduced cardiac output.
  • Option F: Paroxysmal nocturnal dyspnea and dyspnea on exertion are usually the results of congestive heart failure. 

NURSESLABS-SATA-2-051

The nurse is teaching a client about home care and treatment of venous stasis ulcers in his leg. What should be included in the nurse’s instructions? Select all that apply.

  • A. Dressings do not need to be changed frequently because there is minimal drainage.
  • B. Healing will be facilitated by wearing leg compression devices.
  • C. When the client is in a sitting position, he should keep his legs elevated.
  • D. Avoid standing for long periods of time.
  • E. Cool packs can be applied to the ulcers to decrease inflammation.
  • F. Soak the affected extremity in warm water every evening.

Correct Answer: B, C, and D.

Venous stasis occurs when blood flow is decreased, as in immobility, medication therapies and in heart failure. Clients need to self-manage their condition. Early assessment facilitates prompt treatment.

  • Option A: Dressings need to be changed as frequently as necessary because there may be excessive drainage. Avoiding constricting garters or socks with tight bands.Wearing constricting clothing decreases normal blood flow and promotes clotting.
  • Option B: Healing of venous stasis ulcers is dependent on relieving the venous congestion in the extremity. Compression devices and elevation of the extremity are the most effective methods. 
  • Option C: Encourage bedrest and keep the affected leg elevated (depending on size and location of the clot) as indicated. Clients usually require bed rest until symptoms are relieved. The affected leg should be elevated to a position above the heart to decrease swelling.
  • Option D: The client should avoid standing for long periods since this increases venous stasis. Avoiding staying in one position for long periods; get up and move around every hour or so on a long flight. This will avoid the occurrence of venous stasis.
  • Option E: Moist cool packs are NOT used, but moist environment dressings are utilized. Provide warm, moist heat to the affected site. Heat promotes comfort and reduces inflammation.
  • Option F: Wear properly sized, correctly applied compression stockings as indicated. Clients with DVT are at high risk for redevelopment and may need to wear stockings over the long term.

Questions and rationale from Nurseslabs.com Feel free to print or share and link back to us! For more practice questions, please visit our Nursing Test Bank [https://nurseslabs.com/nursing-test-bank]

NURSESLABS-SATA-2-052

A nurse knows the clinical manifestations of a client with Addison’s disease include which of the following? Select all that apply.

  • A. Nausea
  • B. Hypothermia
  • C. Hypertension
  • D. Hyperpigmentation
  • E. Hypotension
  • F. Hypernatremia

Correct Answer: A, D, and E.

Addison’s disease is due to hypofunctioning of the adrenal cortex. The clinical manifestations have a very slow onset, and skin hyperpigmentation is a classic sign. Fatigue, nausea, weight loss, hypotension, hyponatremia, and hyperkalemia are other findings associated with the condition.

  • Option A: Addison disease can occur at any age but most often presents during the second or third decades of life. The initial presentation includes fatigue, generalized weakness, weight loss, nausea, vomiting, abdominal pain, dizziness, tachycardia, and/or postural hypotension.
  • Option B: Due to its variable presentation, a high index of suspicion for Addison disease is necessary when evaluating a conglomeration of non-specific symptoms. These may include unexplained fatigue, poor appetite, chronic abdominal pain, or weight loss.
  • Option D: Hyperpigmentation is characteristic and occurs in almost all patients. It is usually generalized and most prominent in sun-exposed and pressure areas. Elevated ACTH and melanocyte-stimulating hormones are causative factors. It is believed that ACTH binds to the melanocyte receptors, which are responsible for pigmentation.
  • Options C and E: In many cases, the diagnosis is made only after the patient presents with an acute adrenal crisis (hypotension, hyponatremia, hyperkalemia, and hypoglycemia). This may be precipitated by a stressful illness or such triggering factors such as infection, trauma, surgery, vomiting, or diarrhea.
  • Option F: Hyponatremia with or without hyperkalemia and/or hypotension can often be seen in Addison disease. Addisonian crisis is often manifested by severe dehydration, confusion, refractory hypotension, and shock. It is more likely to occur in primary adrenal insufficiency than secondary adrenal insufficiency.

NURSESLABS-SATA-2-053

A nurse is caring for a terminally ill client. The nurse assesses the client for identification of the psychosocial stage of acceptance. What are the five stages of death and dying described by Elisabeth Kübler-Ross? Select all that apply.

  • A. Bargaining
  • B. Anxiety
  • C. Denial and isolation
  • D. Acceptance
  • E. Fear
  • F. Anger
  • G. Depression
  • H. Panic

Correct Answer: A, C, D, F, and G.

Elisabeth Kübler-Ross outlined similarities to psychosocial responses to impending death or loss. The duration during each stage may vary or even overlap from individual to individual. According to Kübler-Ross, the five stages of death and dying are denial and isolation, anger, bargaining, depression, and acceptance.

  • Option A: Bargaining typically manifests as patients seek some measure of control over their illness. The negotiation could be verbalized or internal and could be medical, social, or religious. The patients’ proffered bargains could be rational, such as a commitment to adhere to treatment recommendations or accept help from their caregivers, or could represent more magical thinking, such as with efforts to appease misattributed guilt they may feel is responsible for their diagnosis.
  • Options B, E, and H: Grieving people often feel that they have lost their sense of safety and control in life, and they find themselves panicking or worrying excessively about what or whom else they could lose in the future. They also may have trouble sleeping or taking care of themselves, which can put them at higher risk for anxiety.
  • Option C: Denial is a common defense mechanism used to protect oneself from the hardship of considering an upsetting reality. Kubler-Ross noted that patients would often reject the reality of the new information after the initial shock of receiving a terminal diagnosis.
  • Option D: Acceptance describes recognizing the reality of a difficult diagnosis while no longer protesting or struggling against it. Patients may choose to focus on enjoying the time they have left and reflecting on their memories. They may begin to prepare for death practically by planning their funeral or helping to provide financially or emotionally for their loved ones.
  • Option F: Anger is commonly experienced and expressed by patients as they concede the reality of a terminal illness. It may be directed, as with blame of medical providers for inadequately preventing the illness, of family members for contributing to risks or not being sufficiently supportive, or of spiritual providers or higher powers for the diagnosis’ injustice.
  • Option G: Depression is perhaps the most immediately understandable of Kubler-Ross’s stages, and patients experience it with unsurprising symptoms such as sadness, fatigue, and anhedonia. Spending time in the first three stages is potentially an unconscious effort to protect oneself from this emotional pain.

NURSESLABS-SATA-2-054

A client with chronic renal failure was recently told by the physician of being a poor candidate for a transplant because of chronic uncontrolled hypertension and diabetes mellitus. Now the client tells the nurse, “I want to go off dialysis. I’d rather not live than be on this treatment for the rest of my life.” Which responses are appropriate? Select all that apply.

  • A. Take a seat next to the client and sit quietly to reflect on what was said.
  • B. Say to the client, “We all have days when we don’t feel like going on.”
  • C. Leave the room to allow the client privacy to collect thoughts.
  • D. Say to the client, “You’re feeling upset about the news you got about the transplant.”
  • E. Say to the client, “The treatments are only 3 days a week. You can live with that.”

Correct Answer: A and D.

One of the main ways nurses establish trust with patients is through communication. Because nurses are likely to have the most direct contact with patients, effective nurse-patient communication is critical. Nurses can utilize proven therapeutic communication techniques that promote quality care.

  • Option A: Silence is a therapeutic communication technique that allows the nurse and client to reflect on what has taken place or been said. By waiting quietly and attentively, the nurse encourages the client to initiate and maintain a conversation. 
  • Option B: Using such platitudes as “We all have days when we don’t feel like going on” fails to address the client’s needs. Saying what the client thinks or feels is “good” implies that the opposite is “bad”. Approval, then, tends to limit the client’s freedom to think, speak, or act in a certain way.
  • Option C: The nurse should not leave the client alone abruptly stopping therapeutic communication. Hospital stays can be lonely, stressful times; when nurses offer their time, it shows they value patients and that someone is willing to give them time and attention. Offering to stay for lunch, watch a TV show, or simply sit with patients for a while can help boost their mood.
  • Option D: By reflecting on the client’s implied feelings, the nurse promotes communication. Recognition acknowledges a patient’s behavior and highlights it without giving an overt compliment. 
  • Option E: Reminding the client of the treatment frequency does not address the feelings. Disapproval implies that the client is “wrong”. Consequently, the client feels defensive about his point of view or ideas.

NURSESLABS-SATA-2-055

The nurse is caring for a client whose cultural background is different. Which nursing actions are appropriate? Select all that apply.

  • A. Consider that nonverbal cues, such as eye contact, may have a different meaning in different cultures.
  • B. Respect the client’s cultural beliefs.
  • C. Ask the client if there are cultural or religious requirements that should be considered in the plan of care.
  • D. Explain the nurse’s beliefs so that the client will understand the differences.
  • E. Understand that all cultures experience pain in the same way.

Correct  Answer: A, B, and C.

From triage to discharge, nurses spend an increasingly significant amount of time with their patients, making it critical for them to become culturally competent. Cultural competency in the health care sector supports positive patient outcomes and improves medical research accuracy.

  • Option A: Nonverbal cues may have different meanings in different cultures. In one culture, eye contact may be a sign of disrespect; in another, eye contact may show respect and attentiveness. 
  • Option B: The nurse should always respect the client’s cultural beliefs and ask if there are cultural or religious requirements. This may include food choices or restrictions, body coverings, or time for prayer. 
  • Option C: The nurse should attempt to understand the client’s culture; it is not the client’s responsibility to understand the nurse’s culture. As with any social issue, the first step is awareness. As a nurse, part of the responsibility to a patient is to learn what you can about them.
  • Option D: The nurse should never impose his or her own beliefs on clients. It’s important that nurses avoid making assumptions about cultures they aren’t familiar with. This can lead to a breakdown of trust and rapport between the nurse and their patient and reduce treatment acceptance.
  • Option E: Culture influences a client’s expression of pain. For example, pain may be openly expressed in one culture and quietly endured in another. A cultural group’s expectations and acceptance of pain as a normal part of life will determine whether pain is seen as a clinical problem that requires a clinical solution.
Share to...